SlideShare ist ein Scribd-Unternehmen logo
1 von 64
Contemporary Management of HIV:
When to Start, What to Start
This program is supported by an independent educational grant from
ViiV Healthcare
Slide credit: clinicaloptions.com
About These Slides
 Please feel free to use, update, and share some or all
of these slides in your noncommercial presentations
to colleagues or patients
 When using our slides, please retain the source
attribution:
 These slides may not be published, posted online, or
used in commercial presentations without permission.
Please contact permissions@clinicaloptions.com for
details
Program Director and Core Faculty
Program Chair
Eric S. Daar, MD
Chief, Division of HIV Medicine
Harbor-UCLA Medical Center
Professor of Medicine
David Geffen School of
Medicine at UCLA
Los Angeles, California
Daniel R. Kuritzkes, MD
Chief, Division of Infectious
Diseases
Brigham and Women’s Hospital
Professor of Medicine
Harvard Medical School
Boston, Massachusetts
Faculty Disclosure Information
Eric S. Daar, MD, has disclosed that he has received
consulting fees from AbbVie, Bristol-Myers Squibb, Gilead
Sciences, Janssen, Merck, Teva, and ViiV and funds for
research support from Bristol-Myers Squibb, Gilead
Sciences, Merck, and ViiV.
Daniel R. Kuritzkes, MD, has disclosed that he has received
consulting fees from Gilead Sciences, Merck, and ViiV; fees
for non-CME/CE services received directly from a
commercial interest or their agents (eg, speaker bureaus)
from Gilead Sciences and Merck; and funds for research
support from Gilead Sciences and ViiV.
Case 1:
Patient Newly Diagnosed
With HIV Infection
Case 1: 22-Yr-Old Man Recently
Diagnosed With HIV
 22-yr-old black man presents with newly diagnosed HIV
infection, after testing positive on routine screening
 Tested HIV seronegative 6 mos ago
 No significant medical problems, no medication use
 Initial laboratory studies, including lipids, renal, and
hepatic function are normal
 CD4+ cell count 726 cells/mm3
(40%), HIV-1 RNA
2130 copies/mL
 HIV genotype WT, HBV immune, HCV negative, HLA-
B*5701 negative
 He is unsure if he should start antiretroviral therapy
Case 1: Current Presentation
 Pt expresses concern about having to take medicines
every day and is worried about adverse events
 He is a nonsmoker and has no family history of heart
disease
 He has multiple sex partners, but says he “almost
always” uses condoms when he has sex
Would you recommend antiretroviral
therapy for this pt?
A. Yes
B. No
C. Unsure
 22-yr-old black man with recent HIV seroconversion
 HIV-1 RNA 2130 copies/mL, CD4+ cell count 726 cells/mm3
 No medical problems
 Labs normal, HIV GT WT, HBV immune, HCV negative, HLA-B*5701 negative
 Multiple sex partners
START: Immediate vs Deferred Therapy
for Asymptomatic, ART-Naive Pts
Immediate ART
ART initiated immediately
following randomization
(n = 2326)
INSIGHT START Study Group. N Engl J Med. 2015;373:795-807.
Lundgren J, et al. IAS 2015. Abstract MOSY0302.
Deferred ART
Deferred until CD4+ cell count ≤ 350 cells/mm3
,
AIDS, or event requiring ART
(n = 2359)
HIV-positive, ART-naive
adults with CD4+ cell
count > 500 cells/mm3
(N = 4685)
Study closed by DSMB
following interim analysis
Slide credit: clinicaloptions.com
START: Primary Outcome
Primary Endpoint Immediate ART Deferred ART
No. with event (%) 42 (1.8) 96 (4.1)
Rate/100 PY 0.60 1.38
HR (immediate/deferred) 0.43 (95% CI: 0.30-0.62; P < .001)
 57% reduced risk of
serious events or
death with immediate
ART
 68% of primary
endpoints occurred in
pts with CD4+ cell
counts > 500 cells/mm3
10
8
6
4
2
0
CumulativePercent
WithEvent
0 6 12 18 24 30 36 42 48 54 60
Mos
INSIGHT START Group. N Engl J Med. 2015;373:795-807.
Lundgren J, et al. IAS 2015. Abstract MOSY0302.
2.5
5.3
Immediate ART
Deferred ART
Slide credit: clinicaloptions.com
START: Serious AIDS Events
 72% reduced risk of serious AIDS events with immediate ART
INSIGHT START Study Group. N Engl J Med. 2015;373:795-807.
Lundgren J, et al. IAS 2015. Abstract MOSY0302.
Serious AIDS Events Immediate ART Deferred ART
No. with event (%) 14 50
Rate/100 PY 0.20 0.72
HR (immediate/deferred) 0.28 (95% CI: 0.15-0.50; P < .001)
0 6 12 18 24 30 36 42 48 54 60
Mos
10
8
6
4
2
0
CumulativePercent
WithanEvent
Immediate ART
Deferred ART
Slide credit: clinicaloptions.com
START: Serious Non-AIDS Events
 39% reduced risk of serious non-AIDS events with immediate ART
0 6 12 18 24 30 36 42 48 54 60
Mos
10
8
6
4
2
0
CumulativePercent
WithanEvent
Serious Non-AIDS Events Immediate ART Deferred ART
No. with event (%) 29 47
Rate/100 PY 0.42 0.67
HR (immediate/deferred) 0.61 (95% CI: 0.38-0.97; P = .04)
Immediate ART
Deferred ART
Slide credit: clinicaloptions.com
INSIGHT START Study Group. N Engl J Med. 2015;373:795-807.
Lundgren J, et al. IAS 2015. Abstract MOSY0302.
START: Reduced Risk of Cancers With
Immediate ART
*Immediate ART: squamous cell carcinoma, plasma cell myeloma, bladder cancer, fibrosarcoma.
Deferred ART: gastric adenocarcinoma, breast cancer, ureteric cancer, malignant melanoma, myeloid
leukemia, thyroid cancer, leiomyosarcoma, liver cancer, squamous cell carcinoma of head and neck.
Cancer Event Immediate ART Deferred ART
Kaposi’s sarcoma 1 11
Lymphoma, NHL + HL 3 10
Prostate cancer 2 3
Lung cancer 2 2
Anal cancer 1 2
Cervical or testis cancer 1 2
Other types* 4 9
Total 14 39
INSIGHT START Study Group. N Engl J Med. 2015;373:795-807.
Lundgren J, et al. IAS 2015. Abstract MOSY0302. Slide credit: clinicaloptions.com
START: Adverse Events
 START: No difference in risk of selected adverse events[1,2]
1. INSIGHT START Group. N Engl J Med. 2015;373:795-807.
2. Lundgren J, et al. IAS 2015. Abstract MOSY0302..
Other Secondary
Endpoints[1,2]
Immediate ART
(n = 2326)
Deferred ART
(n = 2359) HR
(95% CI)
P Value
n n/100 PY n n/100 PY
Grade 4 event 73 1.06 73 1.05
1.01
(0.73-1.39)
.97
Unscheduled hospitalization 262 4.02 287 4.40
0.91
(0.77-1.08)
.28
Grade 4 event, unscheduled
hospitalization, or death
from any cause
283 4.36 311 4.78
0.91
(0.77-1.07)
.25
Slide credit: clinicaloptions.com
TEMPRANO: Immediate vs Deferred ART
Initiation and IPT Delivery for African Pts
TEMPRANO ANRS 12136 Study Group. N Engl J Med.
2015;373:808-822.
Mos From Randomization
CumulativeProbability
ofDeathorSevere
HIV-RelatedIllness(%)
25
20
15
10
5
0
0 6 12 18 24 30
Deferred ART
Deferred ART + IPT
Immediate ART
Immediate ART + IPT
30-Mo Probability, %
14.1
8.8
7.4
5.7
Slide credit: clinicaloptions.com
DHHS Recommendations for Early HIV
Infection
 ART recommended for early HIV infection
– Although no definitive data confirming whether this
approach has long-term virologic, immunologic, or
clinical benefits
 ART recommended for pregnant women with early
HIV infection
– To prevent perinatal transmission
 ART can start before drug resistance test results are
available
– Boosted PIs + 2 NRTIs recommended to prevent
resistance in this setting
DHHS Guidelines. November 2015 Slide credit: clinicaloptions.com
HPTN 052: ART for Prevention of HIV
Transmission in Serodiscordant Couples
 International, randomized, controlled trial
Stable, healthy, sexually
active, HIV-discordant
couples with CD4+ cell
count 350-550 cells/mm3
(N = 1763 couples)
Early ART Arm
Initiate ART immediately
(n = 886 couples)
Delayed ART Arm
Initiate ART at CD4+ cell count
≤ 250 cells/mm3
or at development of
AIDS-defining illness
(n = 877 couples)
Cohen MS, et al. IAS 2015. Abstract MOAC0101LB. Slide credit: clinicaloptions.com
HPTN 052: Reduced Risk of Partner
Infection
 ART offered to all index pts
in delayed ART arm from
May 2011 after interim
results
– 84% of pts in delayed ART
arm had initiated ART at
Yr 1 and 98% prior to study
closure
 No linked HIV transmissions
observed when index
participant stably suppressed
on ART
Partner
Infections, n
(rate/100 PY)
Overall
(April 2005 - May 2015)
Early
(4314 PY F/U)
Delayed
(4180 PY F/U)
All 19 (0.44) 59 (1.41)
Linked 3 (0.07) 43 (1.03)
Risk Reduction
With Early ART,
%
All infections 69 --
Linked infections 93 --
Cohen MS, et al. IAS 2015. Abstract MOAC0101LB. Slide credit: clinicaloptions.com
Which regimen would you recommend?
A. Dolutegravir/abacavir/
lamivudine
B. Elvitegravir/cobicistat/
tenofovir DF/emtricitabine
C. Elvitegravir/cobicistat/
tenofovir
alafenamide/emtricitabine
D. Efavirenz/
tenofovir DF/emtricitabine
E. Rilpivirine/
tenofovir DF/emtricitabine
F. Dolutegravir +
tenofovir DF/emtricitabine
G. Raltegravir +
tenofovir DF/emtricitabine
H. Darunavir + ritonavir +
tenofovir DF/emtricitabine
I. I would recommend a
different regimen
 22-yr-old black man with recent HIV seroconversion, multiple sex partners
 HIV-1 RNA 2130 copies/mL, CD4+ cell count 726 cells/mm3
, no medical problems
 Labs normal, HIV GT WT, HBV immune, HCV negative, HLA-B*5701 negative
Discussion Question: Would your choice
change if the pt . . .
 Was HLA-B*5701 positive?
 Was HBsAg positive?
 Had high viral load?
 Was a woman?
 22-yr-old black man with recent HIV seroconversion
 HIV-1 RNA 2130 copies/mL, CD4+ cell count 726 cells/mm3
 No medical problems
 Labs normal, HIV GT WT, HBV immune, HCV negative, HLA-B*5701 negative
 Multiple sex partners
DHHS, IAS-USA, EACS Guidelines:
Recommended Regimens for First-line ART
 Recommendations may differ based on baseline viral load, CD4+ count, CrCl,
eGFR, HLA-B*5701 status, HBsAg status, and osteoporosis status
 Publication of these guidelines preceded the availability of DTG/ABC/3TC as a
single-tablet regimen
1. DHHS Guidelines. November 2015. 2. Günthard HF, et al. JAMA.
2014;312:410-425. 3. EACS Guidelines. October 2015.
Class DHHS[1]
IAS-USA[2]
EACS[3]
INSTI  DTG/ABC/3TC
 DTG + TDF/FTC
 EVG/COBI/TDF/FTC
 EVG/COBI/TAF/FTC
 RAL + TDF/FTC
 DTG + ABC/3TC
 DTG + TDF/FTC
 EVG/COBI/TDF/FTC
 RAL + TDF/FTC
 DTG/ABC/3TC
 DTG + TDF/FTC
 EVG/COBI/TDF/FTC
 RAL + TDF/FTC
Boosted
PI
 DRV + RTV +
TDF/FTC
 DRV + RTV + TDF/FTC
 ATV + RTV + TDF/FTC
 ATV + RTV + ABC/3TC
 DRV + RTV + TDF/FTC
NNRTI  EFV/TDF/FTC
 EFV + ABC/3TC
 RPV/TDF/FTC
 RPV/TDF/FTC
Slide credit: clinicaloptions.com
ACTG 5257: Open-Label ATV + RTV vs
RAL vs DRV + RTV in First-line ART
 Primary endpoints
– Virologic failure: time to HIV-1 RNA > 1000 copies/mL (at Wk 16 or before Wk 24) or
> 200 copies/mL (at or after Wk 24)
– Tolerability failure: time to discontinuation of randomized component for toxicity
 Composite endpoint: the earlier occurrence of either VF or TF in a given participant
 Switch of regimens allowed for tolerability
Lennox JL, et al. Ann Intern Med. 2014;161:461-471.
ART-naive pts with
HIV-1 RNA ≥ 1000
copies/mL
(N = 1809)
ATV + RTV 300 + 100 mg QD +
TDF/FTC
(n = 605)
RAL 400 mg BID +
TDF/FTC
(n = 603)
Stratified by HIV-1 RNA
< or ≥ 100,000 copies/mL, participation in
metabolic substudy, CV risk
DRV + RTV 800/100 mg QD +
TDF/FTC
(n = 601)
Wk 96 after last
pt enrolled
Slide credit: clinicaloptions.com
ACTG 5257: Primary Endpoint Analyses at
Wk 96
 Regimens equivalent
in time to VF
Lennox JL, et al. Ann Intern Med. 2014;161:461-471.
 Significantly greater
incidence of treatment
failure with ATV + RTV
vs RAL or DRV + RTV
– In part due to high
frequency of
hyperbilirubinemia*
 Considering both
efficacy and tolerability,
RAL superior to either
boosted PI
 DRV + RTV superior to
ATV + RTV
Virologic Failure Tolerability Failure Composite Endpoint
Difference in 96-Wk Cumulative Incidence (97.5% CI)
0-10 10 20
ATV + RTV vs RAL
3.4% (-0.7 to 7.4)
DRV + RTV vs RAL
5.6% (1.3-9.9)
ATV + RTV vs DRV + RTV
-2.2% (-6.7 to 2.3)
ATV + RTV vs DRV + RTV
9.2% (5.5-12.9)
0-10 10 20
ATV + RTV vs RAL
12.7% (9.4-16.1)
DRV + RTV vs RAL
3.6% (1.4-5.8)
Favors RAL
Favors DRV + RTV
0-10 10 20
ATV + RTV vs RAL
14.9% (10.2-19.6)
DRV + RTV vs RAL
7.5% (3.2-11.8)
ATV + RTV vs
DRV + RTV
7.5% (2.3-12.7)
Favors RAL
Favors DRV + RTV
Favors RAL
*Pts were allowed to switch regimens and remain on study.
Slide credit: clinicaloptions.com
Study 103: EVG/COBI/TDF/FTC
Noninferior to ATV + RTV + TDF/FTC
Through Wk 144
Outcomes at
Wk 144[3]
EVG/COBI/
TDF/FTC
ATV + RTV +
TDF/FTC
Treatment-
related d/c, %
6 9
Virologic
failure, %
8 7
Mean CD4+
cell count
increase,
cells/mm3
280 293
Clumeck N, et al. J Acquir Immune Defic Syndr. 2014;65:e121-124.
EVG/COBI/TDF/FTC
(n = 353)
ATV + RTV + TDF/FTC
(n = 355)
Δ: 2.7%
(-2.1 to 7.5)
Δ: 1.1%
(-4.5 to 6.7)
Wk 48 Wk 144
78
75
0
20
40
60
80
100
90 87
Δ: 3.1%
(-3.2 to 9.4)
83
82
Wk 96
Slide credit: clinicaloptions.com
WAVES: EVG/COBI/TDF/FTC vs ATV +
RTV + TDF/FTC in Tx-Naive Women
 International, randomized, double-blind phase III trial
 Pts generally well matched at baseline
– Pts with HIV-1 RNA > 100,000 copies/mL: EVG/COBI/TDF/FTC arm 24%;
ATV + RTV + TDF/FTC arm 25%
Squires K, et al. IAS 2015. Abstract MOLBPE08.
EVG/COBI/TDF/FTC QD +
Placebos for ATV, RTV, and TDF/FTC QD
(n = 289)
ATV + RTV + TDF/FTC QD +
Placebo for EVG/COBI/TDF/FTC QD
(n = 286)
HIV-infected women
with HIV-1 RNA
≥ 500 copies/mL;
no previous ART;
and eGFR ≥ 70 mL/min
(N = 575)
Wk 48
Open-label
extension
ATV 300 mg; RTV 100 mg; TDF/FTC 300/200 mg; EVG/COBI/TDF/FTC 150/150/300/200 mg
Slide credit: clinicaloptions.com
WAVES: EVG/COBI/TDF/FTC Superior to
ATV + RTV + TDF/FTC At Wk 48
 No significant differences
between arms in change
from BL for eGFR, spine
or hip BMD, LDL or HDL
cholesterol, total
cholesterol to HDL ratio,
or triglycerides
 Significantly greater
increase in total
cholesterol with
EVG/COBI/ TDF/FTC
 Lower rate of
discontinuations due to
AEs with EVG/COBI/
TDF/FTC vs ATV + RTV
+ TDF/FTC (2.4% vs
7.0%)
Squires K, et al. IAS 2015. Abstract MOLBPE08.
Wk48HIV-1RNA<50c/mL(%)
100
80
60
40
20
0
Overall ≤ 100,000 > 100,000
HIV-1 RNA (copies/mL)
EVG/COBI/TDF/FTC ATV + RTV + TDF/FTC
87
81
86
82
90
78
n = 289 286 220 214 69 72
Emergent Resistance
EVG/COBI/FTC/TDF
(n = 289)
ATV+RTV + TDF/FTC
(n = 286)
Resistance analysis
population 19 21
Developed resistance
mutations to study drugs 0 3
Slide credit: clinicaloptions.com
SINGLE: DTG + ABC/3TC Superior to
EFV/TDF/FTC in Tx-Naive Pts Through Wk 144
 Emergent resistance in those with VF: 0/39 (DTG) vs 7/33 (EFV)
Virologic
Success*[1]
Virologic
Nonresponse[2]
No Virologic
Data[2]
Pts(%)
Favors
EFV/TDF/FTC
95% CI for Difference†
0%
Wk 48
Wk 96
Wk 144
7.4%
8.0%
8.3%
2.5%
2.3
%
2% 14.6%
13.8%
12.3%
Favors
DTG+ABC/3TC
15%
1. Walmsley S, et al. J Acquir Immune Defic Syndr. 2015;70:515-519.
2. Pappa K, et al. ICAAC 2014. Abstract H-647a.
88
81 80
72 71
63
5 6 7 8 10
7 7
13 12
20
30
18
100
80
60
40
20
0
DTG + ABC/3TC QD (n = 414)
EFV/TDF/FTC QD (n = 419)
Wk 48 96 144 Wk 48 96 144 Wk 48 96 144
*HIV-1 RNA < 50 copies/mL as defined by FDA Snapshot algorithm.
†
-10% noninferiority margin.
-5%
Slide credit: clinicaloptions.com
SPRING-2: DTG + 2 NRTIs Noninferior to
RAL + 2 NRTIs Through Wk 96
Outcomes at
Wk 96[2] DTG + NRTIs RAL + NRTIs
D/c for AEs or
death, %
2 2
Virologic
nonresponse, %
5 10
Mean CD4+ cell
count increase,
cells/mm3
276 264
HIV-1RNA<50copies/mL(%)
88 85
DTG 50 mg QD
(n = 411)
RAL 400 mg BID
(n = 411)
0
20
40
60
80
100
81
76
Wk 48[1]
Wk 96[2]
1. Raffi F, et al. Lancet. 2013;381:735-743.
2. Raffi F, et al. Lancet Infect Dis. 2013;13:927-935.
361/
411
351/
411
332/
411
314/
411
Δ 4.5%
(-1.1% to 10.0%)
Δ 2.5%
(-2.2% to 7.1%)
n/N =
Slide credit: clinicaloptions.com
40
FLAMINGO: DTG Superior to DRV + RTV
in ART-Naive Pts Through Wk 96
Virologic Success Virologic Nonresponse No Virologic Data
Favors
DRV + RTV
95% CI for Difference
0%-12%
Wk 48
Wk 96
Subjects(%)
Favors
DTG
25%
DTG + 2 NRTIs (n = 242)
DRV + RTV + 2 NRTIs (n =
242)
Molina J-M, et al. Lancet HIV 2015;2:e127–36.
7.1%
12.4%
0.9%
4.7% 20.2%
13.2%
Wk 48 Wk 96 Wk 48 Wk 96 Wk 48 Wk 96
100
80
60
20
0
83
90
80
68
6 7 8
12
4
10 12
21
15 18 19 28 10 24 29 50217 200 194 164n =
Slide credit: clinicaloptions.com
Studies 104/111: Tenofovir Alafenamide
Fumarate vs TDF in Treatment-Naive Pts
 Parallel, randomized, double-blind, active-controlled phase III studies
 Primary endpoint: HIV-1 RNA < 50 c/mL at Wk 48, as defined by FDA
Snapshot algorithm
EVG/COBI/FTC/TAF*
single-tablet regimen
(n = 866)
EVG/COBI/FTC/TDF†
single-tablet regimen
(n = 867)
Treatment-naive
HIV-infected pts with
HIV-1 RNA ≥ 1000 copies/mL,
eGFR ≥ 50 mL/min
(N = 1733)
Stratified by HIV-1 RNA,
CD4+ cell count, geographic region
Wk 48
Primary endpoint Wk 144
*150/150/200/10 mg once daily.
†
150/150/200/300 mg once daily.
Sax PE, et al. Lancet. 2015;385:2606-2615. Slide credit: clinicaloptions.com
Studies 104/111: TAF Noninferior to TDF
at Wk 48
 EVG/COBI/FTC/TAF was noninferior to EVG/COBI/FTC/TDF at Wk 48 in each study:
93% vs 92% (Study 104); 92% vs 89% (Study 111)
– Race not significant predictor of virologic efficacy in multivariate analysis
 Declines in eGFR and in hip and spine BMD significantly less in TAF arm

Discontinued for AE, death, or missing data.
1. Sax PE, et al. Lancet. 2015;385:2606-2615.
2. Wohl D, et al. ID Week 2015. Abstract 1073.
No Data
Virologic
Success
Virologic
Failure
Pts(%)
92
90
EVG/COBI/FTC/TAF
(n = 866)
EVG/COBI/FTC/TDF
(n = 867)
0
20
40
60
80
100
4 4 4 6
n = 800 784
Favors TAF
0
4.7%-0.7%
2.0%
Treatment Difference (95% CI)
-12% +12%
Favors TDF
Virologic Outcome
Slide credit: clinicaloptions.com
Studies 104/111: Renal Outcomes With
TAF vs TDF in Black vs Nonblack Pts
 In black pts, decrease in median eGFR significantly smaller with
TAF vs TDF
 Less spine and hip BMD loss with TAF vs TDF both in black
and nonblack pts
Wohl D, et al. ID Week 2015. Abstract 1073.
Black Pts
EVG/COBI/FTC/TAF
Nonblack Pts
EVG/COBI/FTC/TDF
15
10
5
0
-5
-10
MeaneGFRChange(mL/min)
-15
-20
-25
-30
15
10
5
0
-5
-10
-15
-20
-25
-30
0 2 4 8 12 16 24 36 48
Wk
0 2 4 8 12 16 24 36 48
Wk
Slide credit: clinicaloptions.com
Potential Advantages and Disadvantages
of Single-Tablet Regimens
Advantages Disadvantages
 Simplicity
 Convenience
 Fewer copays
 Reduces selective nonadherence to
components of regimen
 Inability to adjust dosages of
components if needed due to drug–
drug interactions or tolerability
issues, eg, renal insufficiency
 Not available for all ART regimens
 Not available for all NRTI pairings
Slide credit: clinicaloptions.com
Available Single-Tablet Regimens
Agent Type Year of FDA
Approval
Efavirenz/tenofovir DF/
emtricitabine (EFV/TDF/FTC)
NNRTI + dual NRTI 2006
Rilpivirine/tenofovir DF/
emtricitabine (RPV/TDF/FTC)
NNRTI + dual NRTI 2011
Elvitegravir/cobicistat/
tenofovir DF/emtricitabine
(EVG/COBI/TDF/FTC)*
INSTI + booster + dual NRTI 2012
Dolutegravir/abacavir/lamivudine
(DTG/ABC/3TC)*
INSTI + dual NRTI 2014
Elvitegravir/cobicistat/
tenofovir alafenamide/emtricitabine
(EVG/COBI/TAF/FTC)*
INSTI + booster + dual NRTI 2015
*DHHS recommended regimen for initial ART.
Slide credit: clinicaloptions.com
Take-Home Points
 Randomized trial data support ART initiation in pts
with CD4+ cell count > 500 cells/mm3
 ART guidelines recommend ART for all pts regardless
of CD4+ cell count
 Recommended regimens for ART initiation have been
revised
– NNRTIs removed from DHHS first-line options
– EVG/COBI/TAF/FTC added to DHHS first-line options,
safe in pts with CrCl ≥ 30 mL/min
Slide credit: clinicaloptions.com
Case 2:
HIV-Infected Patient
With Common Comorbidities
Case 2: 53-Yr-Old Man With HIV Infection
and Multiple Medical Problems
 53-yr-old man presents with newly diagnosed HIV
infection
 Tested for HIV infection by his PCP, who has been
treating him for hyperlipidemia and DM for 12 yrs
 Hospitalized 2 yrs earlier for chest pain and
diagnosed with NSTEMI
 Reports he has been better in the last yr at sticking to
his medical regimen and now rarely misses a dose of
his prescribed medications (metformin, glipizide,
aspirin, metoprolol, and atorvastatin)
 He is an exsmoker and denies use of illicit drugs
Slide credit: clinicaloptions.com
Case 2: Laboratory Analysis
 CD4+ count 423 cells/mm3
(27%), HIV-1 RNA 69,554
copies/mL
 HIV genotype K103N (resistant to EFV, NVP, DLV)
 HLA-B*5701 negative
 Cr/BUN 1.6/20, eGFRCG 53 mL/min
 ALT/AST normal, HBV immune, HCV negative
 Last recorded A1c 6.5; last LDL 105 mg/dL
 His hyperlipidemia and DM are relatively well
controlled
 He is interested in starting ART Slide credit: clinicaloptions.com
Which NRTI combination would you
recommend?
A. Abacavir/lamivudine
B. Tenofovir DF/emtricitabine
C. Tenofovir alafenamide/emtricitabine (as E/C/F/TAF)
D. I would use a different NRTI combination
E. I would use lamivudine or emtricitabine without other NRTIs
F. I would not use NRTIs in this pt
G. Unsure
 53-yr-old man recently diagnosed with HIV infection
 HIV-1 RNA 69,554 copies/mL, CD4+ count 423 cells/mm3
 Hyperlipidemia and DM controlled on medication, history of NSTEMI
 Cr/BUN 1.6/20, eGFR 53 mL/min, A1c 6.5, LDL 105 mg/dL
 HIV GT: K103N, HBV immune, HCV negative, HLA-B*5701 negative
Which other agents would you use in the
regimen?
A. Boosted PI
B. INSTI
C. Boosted PI + INSTI
D. Other
E. Unsure
 53-yr-old man recently diagnosed with HIV infection
 HIV-1 RNA 69,554 copies/mL, CD4+ count 423 cells/mm3
 Hyperlipidemia and DM controlled on medication, history of NSTEMI
 Cr/BUN 1.6/20, eGFR 53 mL/min, A1c 6.5, LDL 105 mg/dL
 HIV GT: K103N, HBV immune, HCV negative, HLA-B*5701 negative
Rising Rates of Comorbidities at HIV
Diagnosis in USA
Pts(%)
Meyer N, et al. IAS 2015. Abstract MOPEB157.
Medicare (> 65 Yrs)
100
80
60
40
20
0
All CV HTN DM Renal
2003 (n = 177; mean age: 72.2 yrs)
2013 (n = 436; mean age: 72.9 yrs)
Pts(%)
Medicaid
100
80
60
40
20
0
All CV HTN DM Renal
2003 (n = 3008; mean age: 34.7 yrs)
2013 (n = 1632; mean age: 39.2 yrs)
Slide credit: clinicaloptions.com
IAS-USA: Recommendations for Initial
ART in the Settings of Specific Conditions
 In pts with or at high risk of CVD, consider avoiding
ABC, LPV/RTV, or FPV + RTV
 In pts with reduced renal function, TDF should
generally be avoided, especially with a boosted PI
 In pts at elevated fracture risk (eg, HCV coinfection,
postmenopausal women, osteoporosis), it may be
prudent to avoid TDF, especially with a boosted PI
Günthard HF, et al. JAMA. 2014;312:410-425. Slide credit: clinicaloptions.com
1.80
1.60
1.40
1.20
1.00
0.00
D:A:D: Cumulative Exposure to ARVs
Associated With Increased CKD Risk
CKD Risk by Yrs of ARV Exposure, IRR
(95% CI)
Drug 1 Yr 2 Yrs 5 Yrs
TDF
1.12
(1.06-1.18)
1.25
(1.12-1.39)
1.74
(1.33-2.27)
ATV+
RTV
1.27
(1.18-1.36)
1.61
(1.40-1.84)
3.27
(2.32-4.61)
LPV/
RTV
1.16
(1.10-1.22)
1.35
(1.21-1.50)
2.11
(1.62-2.75)
Mocroft A, et al. CROI 2015. Abstract 142.
Relationship Between Increasing
Exposure to ARVs and CKD
ATV + RTV LPV/RTVTDF
Multivariate (on treatment)
Multivariate (TDF censored)
Univariate
Multivariate
Slide credit: clinicaloptions.com
Summary of Key Analyses Showing ABC
Associated With Risk of MI
Study
Study
Design
Age, Yrs
(Range)
Event
(n)
Pts,
N
TDF
CV
Effect
ABC
CV
Effect
Time on
ABC, Mos
Risk of MI
(95% CI)
D:A:D[1]
Cohort
40
(35-47)
MI, validated
(387)
22,625 No Yes ≥ 6
2.04
(1.66-2.51)
D:A:D 2015[2]
Cohort
39
(33-46)
MI
(493)
32,663 Yes Current
1.47
(1.26-1.71)
SMART[3]
RCT
45
(39-51)
MI, validated
(19)
2752 No Yes Current
4.3
(1.4-13.0)
STEAL[4]
RCT
45.7
±8.8
MI
(4)
357 No Yes 96
2.79*
(1.76-4.43)
QPHID[5]
CC
47
(22-67)
MI
(125)
7053 No Yes Any
1.79
(1.16-2.76)
Danish[6]
Cohort
39
(33-47)
MI
(67)
2952
No Yes > 6
2.00
(1.07-3.76)
VA (Choi)[7]
Cohort 46
CVD event
(501)
10,931 No Yes Recent
1.64
(0.88-3.08)
Swiss[8]
Cohort
Not
given
CVD event
(365)
11,856 No Yes Recent
4.06†
(2.24-7.34)
MAGNIFICENT[9]
CC
50
(22-85.5)
CVD event
(571)
1875 No Yes Current
1.56
(1.17-2.07)
NA-ACCORD[10]
Cohort
MI, validated
(301)
16,733 Yes ≤ 6 1.33
References in slidenotes Slide credit: clinicaloptions.com
*Risk for serious non-AIDS events (most common was CVD, including MI); HR for CVD with TDF
vs ABC: 0.12 (95% CI: 0.02-0.98; P = .048).
†
Risk for CVD event, including MI, invasive CV procedure, or CV-related death.
Summary of Key Analyses Showing ABC
NOT Associated With Risk of MI
References in slidenotes
Study Study
Design
Age, Yrs
(Range)
Event
(n)
Pts,
N
TDF
CV
Effect
ABC
CV
Effect
Time
on ABC,
Mos
Adj Risk of
MI
(95% CI)
FHDH[1]
CC
47
(41-54)
MI
(289)
74,958 No No
< 12/
recent
1.27*
(0.64-2.49)
ALLRT/
ACTG[2] Cohort
37
(26-51)
MI
(36)
5056 No No 72
0.6
(0.3 -1.4)
VA[3]
Cohort 46
MI
(278) 19,424
No No Per 12
1.18
(0.92-1.50)
FDA[4]
Meta-
analysis
of RCTs
36-42
MI
(46)
9868 No No 19
1.02
(0.56-1.84)
NA-ACCORD[5]
Cohort
MI,
validated
(301)
16,733 No ≤ 6 1.33
*Without adjustment for cocaine use OR: 2.01 (1.11-3.64).
Slide credit: clinicaloptions.com
Subanalysis of 104/111: TAF Noninferior to
TDF at Wk 48 in Pts Aged 50 Yrs or Older
 Mean eGFR decline and mean % BMD decrease from baseline to Wk 48 significantly
lower with TAF vs TDF (eGFR: P = .01; spine: P = .01; hip: P < .001)
Daar ES, et al. ID Week 2015. Abstract 1074.
100
80
60
40
20
0
Virologic
Success
Virologic
Failure
No Data
EVG/COBI/FTC/TAF
(n = 89)
EVG/COBI/FTC/TDF
(n = 114)
94 91
3 4 2 4
-14 -10 -6 -2 2 6 10 14
-5.2
3.5
12.2
Favors TAFFavors TDF
Treatment Difference (95% CI)Virologic Outcome
Pts(%)
Slide credit: clinicaloptions.com
Phase III Trial of Stable Switch to
EVG/COBI/TAF/FTC in Pts With CKD
 Multicenter, single-arm, phase III switch study
 Primary safety endpoint: eGFR at Wk 24
 Baseline characteristics
– Median age 58 yrs, median eGFR 56 mL/min, clinically significant
albuminuria 49%, median CD4+ count 632 cells/mm3
, pre-switch TDF
65%, HTN 40%, DM 14%
EVG/COBI/TAF/FTC (150/150/10/200 mg QD)
single-tablet regimen
HIV-infected pts with
HIV-1 RNA < 50 copies/mL
for ≥ 6 mos, CD4+ cell count
≥ 50 cells/mm3
, and
eGFR 30-69 mL/min
(N = 242)
Wk 24
Primary endpoint Wk 96
Gupta S, et al. IAS 2015. Abstract TUAB0103. Slide credit: clinicaloptions.com
Change in Renal Function Following
Switch to EVG/COBI/TAF/FTC
 No change in actual GFR at Wk 24
 In pts on TDF, tubular proteinuria improved after switch
MedianChangeFrom
Baseline
eGFRCG
mL/min
eGFRCKD-EPI Cr
mL/min/1.73m2
eGFRCKD-EPI cys C
mL/min/1.73m2
TDF at BL Non-TDF at BLAll pts
-0.6
+0.2
-1.8 -1.8* -1.5
-2.7*
+1.6*
-1.4
+2.7**P < .05
Gupta S, et al. IAS 2015. Abstract TUAB0103.
Change in eGFR From Baseline to Wk 48
10
0
-10
Slide credit: clinicaloptions.com
Considerations for Pts With Renal
Impairment
CrCl (mL/min) EVG/COBI/FTC/TAF[1]
EVG/COBI/FTC/TDF[2]
≥ 70 No adjustment needed No adjustment needed
50-70 No adjustment needed Initiation not recommended
30-49 No adjustment needed Initiation not recommended;
Discontinue if CrCl declines to this
level during treatment
< 30 Initiation not recommended Initiation not recommended
Discontinue if CrCl declines to this
level during treatment
1. EVG/COBI/FTC/TAF [package insert].
2. EVG/COBI/FTC/TDF [package insert]. Slide credit: clinicaloptions.com
If you were considering an ABC, TDF, and TAF–
sparing regimen, which would you most
strongly consider?
A. Boosted PI monotherapy
B. Boosted PI + INSTI ± 3TC (or FTC)
C. Boosted PI + 3TC (or FTC)
D. DTG + 3TC
E. I would not consider an ABC-, TDF-, and TAF-sparing regimen
in this type of pt
F. Other
 53-yr-old man recently diagnosed with HIV infection
 HIV-1 RNA 69,554 copies/mL, CD4+ count 423 cells/mm3
 Hyperlipidemia and DM controlled on medication, history of NSTEMI
 Cr/BUN 1.6/20, eGFR 53 mL/min, A1c 6.5, LDL 105 mg/dL
 HIV GT: K103N, HBV immune, HCV negative, HLA-B*5701 negative
< 100,000 c/mL
≥ 100,000 c/mL
n = 530
n = 275
BL HIV-1 RNA
NEAT: RAL + DRV/RTV Noninferior to
TDF/FTC + DRV/RTV in Naive Pts at 96 Wks
 Randomized, open-label phase III study of DRV/RTV + RAL vs
DRV/RTV + TDF/FTC in ART-naive pts
Raffi F, et al. Lancet. 2014;384:1942-1951.
Overall N = 805
Primary Endpoint at Wk 96: Adjusted Difference in Proportion
of Pts With Failure (RAL - TDF/FTC [95% CI])
-10 0 10 20 30
RAL TDF/FTC Adjusted Difference
Estimate (95% CI)
17.8 13.8 4.0 (-0.8 to 8.8)
7.4
36.8
7.3
27.3
0.1 (-3.8 to 4.0)
9.6 (-0.1 to 20.1)
43.2
13.7
20.9
12.3
22.3 (7.4 to 37.1)
1.4 (-3.5 to 6.3)
< 200/mm3
≥ 200/mm3
n = 123
n = 682
BL CD4+ cell count
Slide credit: clinicaloptions.com
GARDEL: Dual ART Noninferior to Triple
ART in Tx-Naive Pts at Wks 48 and 96
 Phase III, international, open-label, randomized study
 Safety and tolerability also similar between treatment arms
Virologic
Success
Virologic
Nonresponse
D/C due to AE
or Death
D/C for Other
Reasons
1. Cahn P, et al. EACS 2015. Abstract 961.
2. Cahn P, et al. Lancet Infect Dis. 2014;14:572-80.
Wk 48 difference: +4.6%
(95% CI: -2.2 to 11.8; P = .171)
Wk 96 difference: +5.9%
(95% CI: -2.3 to 14.1; P = .165)
100
80
60
40
20
0
Pts(%)
LPV/RTV + 3TC
LPV/RTV + 3TC
or FTC + NRTI
4.7 5.9 3 5 6 82.4 2.1
0.6
2.8 6.6 10.6
88.3
83.7
90.3
84.4
Wk: 96[1]
48[1]
96[1]
48[2]
96[1]
48[2]
96[1]
48[2]
Slide credit: clinicaloptions.com
PADDLE: Dolutegravir + Lamivudine in
Treatment-Naive Pts
 Open-label, single-arm phase IV exploratory trial
 BL RNA: median 24,128 copies/mL; IQR 11,686-36,794
copies/mL
 20 of 20 pts met primary endpoint of HIV-1 RNA < 50 copies/mL
at Wk 24 (ITT-e, FDA snapshot analysis)
– Including 4 pts with BL HIV-1 RNA > 100,000 copies/mL
– All pts virologically suppressed by Wk 8
Treatment-naive pts
with HIV-1 RNA
5000-100,000 copies/mL;
CD4+ ≥ 200 cells/mm3
;
HBsAg negative
(N = 20)
DTG 50 mg QD + 3TC 300 mg QD
(N = 20*)
*Pts enrolled in 2 cohorts of 10 pts. Second cohort enrolled following
confirmation of first cohort success at Wk 8.
Figueroa MI, et al. EACS 2015. Abstract 1066. Slide credit: clinicaloptions.com
Recommendations on the Use of NRTI-
Sparing Regimens in First-line ART
 Regimens using < 2 NRTIs should only be used in pts
who cannot take ABC or TDF
 These regimens can be considered when ABC or
TDF cannot be used:
– DRV + RTV + RAL (only for pts with HIV-1 RNA
< 100,000 copies/mL and CD4+ cell count > 200
cells/mm3
)
– LPV/RTV (BID) + 3TC (BID)
DHHS Guidelines. November 2015 Slide credit: clinicaloptions.com
Case 3:
Patient With
Advanced HIV Disease
Case 3: 40-Yr-Old Woman With An OI
 A 40-yr-old woman presents to the ED with L arm
weakness, difficulty walking, and dysarthric speech
 Prior history of positive HIV serology, but she is
homeless and has neither entered into care for HIV
nor received ART
 CT scan shows R frontoparietal mass with extensive
edema; no hydrocephalus or midline shift
 Toxoplasma IgG is positive
 Pt is started on sulfadiazine, pyrimethamine and
folinic acid with improvement in her neurological
status
Slide credit: clinicaloptions.com
Case 3, Laboratory Analysis
 HIV-1 RNA 175,000 copies/mL, CD4+ count 65 cells/mm3
(4%)
 HIV genotype is pending
 Coinfection with HCV (HCV RNA positive) and HBV
(HBsAg positive)
 HLA-B*5701 negative
 No known history of heart disease or renal insufficiency
 History of injection drug use
 She smokes 1 pack/day and uses methamphetamine and
opiates when she can get them
Slide credit: clinicaloptions.com
When would you recommend starting
antiretroviral therapy?
A. Start ART within 2-3 weeks
B. Defer ART until the pt completes treatment for
toxoplasmic encephalitis
C. Defer ART until the pt is in a stable living situation
D. Defer ART until the pt starts a drug treatment program
E. Unsure
 40-yr-old woman with untreated HIV infection and new dx of toxoplasmic encephalitis
 HIV-1 RNA 175,000 copies/mL, CD4+ cell count 65 cells/mm3
 HIV GT pending, HBsAg positive, HCV RNA positive, HLA-B*5701 negative
 History of injection drug use
 Homeless
Favors Deferred ART
Zolopa AR, et al. PLoS ONE. 2009;4:e5575.
ACTG 5164: Immediate vs Deferred ART in
Pts With Acute Opportunistic Infections
Risk of AIDS Progression/Death by Entry Diagnoses, Log OR (95% CI)
Total
PCP
Bacterial infection
Other OI*
Fungal
Crypto
Mycobacterial
> 1 OI
CD4+ < 50
CD4+ ≥ 50
Events, n/N
54/282
28/181
11/41
42/194
12/52
8/41
8/18
30/148
39/196
15/86
0 0.25 0.5 1.0 8.0 202.5
Favors Early ART
*Includes 13 pts
with toxoplasmosis
Slide credit: clinicaloptions.com
Recommendations for ART in Pts With
Selected Opportunistic Infections
Opportunistic Infection DHHS Recommendation for ART
Pneumocystis pneumonia  Start ART within 2 wks of PCP diagnosis
Toxoplasma gondii encephalitis  Many clinicians start ART within 2-3 wks
 Based on A5164 study, in which the 282 pts
with OIs included 13 pts (5%) with
toxoplasmosis
Mycobacterium tuberculosis  Start ART within 2 wks if CD4+ < 50
cells/mm3
, by 8-12 wks for all others
 Consider DDIs, adherence support
Cryptosporidiosis  Start ART as part of OI management
Cryptococcal meningitis  Consider delaying ART until after
antifungal induction (2 wks) or
induction/consolidation (10 wks)
DHHS Guidelines. November 2015 Slide credit: clinicaloptions.com
Which regimen would you recommend if
starting ART prior to discharge?
A. Dolutegravir/abacavir/lamivudine
B. Dolutegravir + tenofovir DF/emtricitabine
C. Elvitegravir/cobicistat/tenofovir DF/emtricitabine
D. Elvitegravir/cobicistat/tenofovir alafenamide/emtricitabine
E. Raltegravir + tenofovir DF/emtricitabine
F. Boosted darunavir + tenofovir DF/emtricitabine
G. NNRTI-based regimen
H. Something else
 40-yr-old woman with untreated HIV infection and new dx of toxoplasmic encephalitis
 HIV-1 RNA 175,000 copies/mL, CD4+ cell count 65 cells/mm3
 HIV GT pending, HBsAg positive, HCV RNA positive, HLA-B*5701 negative
 History of injection drug use
 Homeless
Prevalence of Drug Resistance Mutations
in Treatment-Naive Pts, 2000-2013
 Baseline plasma samples from
4 phase III trials (GS 903, 934,
104, 111; N = 2531)
– 1617 samples analyzed for
integrase mutations
– 2531 analyzed for protease or
RT mutations
 Substantial ↑ in prevalence of
NNRTI resistance, modest ↑ in
PI resistance
 Stable prevalence of NRTI
resistance (mostly TAMs)
– M184V/I ≤ 0.2%; K65R ≤ 0.1%
 Little evidence of transmitted
INSTI resistance over period
– Mostly T97A polymorphism
2000 (GS-903)
2003 (GS-934)
2013 (GS-104/GS-
111)
0
2
NNRTI
10
4
6
8
NRTI PI INSTI
0.5 1.0
0
4.2
8.7
3.2
2.6 2.6
1.2
2.4
2.9
1.4
Margot NA, et al. CROI 2014. Abstract 578. Slide credit: clinicaloptions.com
Summary
 Randomized trial data support ART initiation in pts
with CD4+ cell count > 500 cells/mm3
 ART guidelines recommend ART for all pts regardless
of CD4+ cell count
 Recommended regimens for ART initiation have been
revised
 ART selection should be individualized according to
pt requirements, the evidence base, and practice
guidance
Slide credit: clinicaloptions.com
Go Online for More CCO
Coverage of HIV!
Additional slidesets on contemporary management of HIV with expert
faculty commentary
Postconference Clinical Updates available following CROI, the
International AIDS Conference, and IDWeek
clinicaloptions.com/hiv

Weitere ähnliche Inhalte

Was ist angesagt?

Contemporary Management of HIV.How Aging Affects ART Management.2018
Contemporary Management of HIV.How Aging Affects ART Management.2018Contemporary Management of HIV.How Aging Affects ART Management.2018
Contemporary Management of HIV.How Aging Affects ART Management.2018hivlifeinfo
 
Современное лечение ВИЧ.Усилить или не усилить : преимущества и недостатки бу...
Современное лечение ВИЧ.Усилить или не усилить : преимущества и недостатки бу...Современное лечение ВИЧ.Усилить или не усилить : преимущества и недостатки бу...
Современное лечение ВИЧ.Усилить или не усилить : преимущества и недостатки бу...hivlifeinfo
 
Современное лечение ВИЧ: индивидуализация стартовой АРТ /Contemporary Manage...
Современное лечение ВИЧ:  индивидуализация стартовой АРТ /Contemporary Manage...Современное лечение ВИЧ:  индивидуализация стартовой АРТ /Contemporary Manage...
Современное лечение ВИЧ: индивидуализация стартовой АРТ /Contemporary Manage...hivlifeinfo
 
Современное лечение ВИЧ: модификация АРТ у пациентов с вирусной супрессией и ...
Современное лечение ВИЧ: модификация АРТ у пациентов с вирусной супрессией и ...Современное лечение ВИЧ: модификация АРТ у пациентов с вирусной супрессией и ...
Современное лечение ВИЧ: модификация АРТ у пациентов с вирусной супрессией и ...hivlifeinfo
 
Антиретровирусные средства и хроническая болезнь почек.Exposure to antiretrov...
Антиретровирусные средства и хроническая болезнь почек.Exposure to antiretrov...Антиретровирусные средства и хроническая болезнь почек.Exposure to antiretrov...
Антиретровирусные средства и хроническая болезнь почек.Exposure to antiretrov...hivlifeinfo
 
Современное лечение ВИЧ: новые парадигмы в АРТ / Contemporary Management of H...
Современное лечение ВИЧ: новые парадигмы в АРТ / Contemporary Management of H...Современное лечение ВИЧ: новые парадигмы в АРТ / Contemporary Management of H...
Современное лечение ВИЧ: новые парадигмы в АРТ / Contemporary Management of H...hivlifeinfo
 
Contemporary Management of HIV.How Common Comorbidities Affect ART Management...
Contemporary Management of HIV.How Common Comorbidities Affect ART Management...Contemporary Management of HIV.How Common Comorbidities Affect ART Management...
Contemporary Management of HIV.How Common Comorbidities Affect ART Management...hivlifeinfo
 
Highlights of AIDS 2016
Highlights of AIDS 2016Highlights of AIDS 2016
Highlights of AIDS 2016hivlifeinfo
 
Key Slides on Individualizing ART Management Based on Treatment Safety and To...
Key Slides on Individualizing ART Management Based on Treatment Safety and To...Key Slides on Individualizing ART Management Based on Treatment Safety and To...
Key Slides on Individualizing ART Management Based on Treatment Safety and To...hivlifeinfo
 
Сравнение режимов лечения ВИЧ в разрезе различных клинических сценариев.ART...
Сравнение  режимов лечения ВИЧ в  разрезе различных клинических сценариев.ART...Сравнение  режимов лечения ВИЧ в  разрезе различных клинических сценариев.ART...
Сравнение режимов лечения ВИЧ в разрезе различных клинических сценариев.ART...hivlifeinfo
 
Современное лечение ВИЧ : лечение возрастных пациентов.2017/Contemporary Mana...
Современное лечение ВИЧ : лечение возрастных пациентов.2017/Contemporary Mana...Современное лечение ВИЧ : лечение возрастных пациентов.2017/Contemporary Mana...
Современное лечение ВИЧ : лечение возрастных пациентов.2017/Contemporary Mana...hivlifeinfo
 
Современное лечение ВИЧ : АРТ как профилактика.Contemporary Management of HIV...
Современное лечение ВИЧ : АРТ как профилактика.Contemporary Management of HIV...Современное лечение ВИЧ : АРТ как профилактика.Contemporary Management of HIV...
Современное лечение ВИЧ : АРТ как профилактика.Contemporary Management of HIV...hivlifeinfo
 
HIV Alert:ART Considerations for Aging Patients.2018
HIV Alert:ART Considerations for Aging Patients.2018HIV Alert:ART Considerations for Aging Patients.2018
HIV Alert:ART Considerations for Aging Patients.2018hivlifeinfo
 
Современное лечение ВИЧ: лечение многократно леченных пациентов с резистентно...
Современное лечение ВИЧ: лечение многократно леченных пациентов с резистентно...Современное лечение ВИЧ: лечение многократно леченных пациентов с резистентно...
Современное лечение ВИЧ: лечение многократно леченных пациентов с резистентно...hivlifeinfo
 
Innovative Paradigms for ART.2019
Innovative Paradigms for ART.2019Innovative Paradigms for ART.2019
Innovative Paradigms for ART.2019hivlifeinfo
 
Why, when, and how to use pre exposure prophylaxis for hiv acquisition. 2014
Why, when, and how to use pre exposure prophylaxis for hiv acquisition. 2014Why, when, and how to use pre exposure prophylaxis for hiv acquisition. 2014
Why, when, and how to use pre exposure prophylaxis for hiv acquisition. 2014Hivlife Info
 
Современное лечение ВИЧ: лечение ВИЧ у пациентов с вирусными гепатитами.Conte...
Современное лечение ВИЧ: лечение ВИЧ у пациентов с вирусными гепатитами.Conte...Современное лечение ВИЧ: лечение ВИЧ у пациентов с вирусными гепатитами.Conte...
Современное лечение ВИЧ: лечение ВИЧ у пациентов с вирусными гепатитами.Conte...hivlifeinfo
 
Determining Candidacy and Strategies for ART Modification.2019
Determining Candidacy and Strategies for ART Modification.2019Determining Candidacy and Strategies for ART Modification.2019
Determining Candidacy and Strategies for ART Modification.2019hivlifeinfo
 
АРТ в 2016-2017 гг: неизменная потребность в индивидуализации лечения для улу...
АРТ в 2016-2017 гг: неизменная потребность в индивидуализации лечения для улу...АРТ в 2016-2017 гг: неизменная потребность в индивидуализации лечения для улу...
АРТ в 2016-2017 гг: неизменная потребность в индивидуализации лечения для улу...hivlifeinfo
 
Incorporating New ART Options Into First-line and Switch Strategies for HIV C...
Incorporating New ART Options Into First-line and Switch Strategies for HIV C...Incorporating New ART Options Into First-line and Switch Strategies for HIV C...
Incorporating New ART Options Into First-line and Switch Strategies for HIV C...hivlifeinfo
 

Was ist angesagt? (20)

Contemporary Management of HIV.How Aging Affects ART Management.2018
Contemporary Management of HIV.How Aging Affects ART Management.2018Contemporary Management of HIV.How Aging Affects ART Management.2018
Contemporary Management of HIV.How Aging Affects ART Management.2018
 
Современное лечение ВИЧ.Усилить или не усилить : преимущества и недостатки бу...
Современное лечение ВИЧ.Усилить или не усилить : преимущества и недостатки бу...Современное лечение ВИЧ.Усилить или не усилить : преимущества и недостатки бу...
Современное лечение ВИЧ.Усилить или не усилить : преимущества и недостатки бу...
 
Современное лечение ВИЧ: индивидуализация стартовой АРТ /Contemporary Manage...
Современное лечение ВИЧ:  индивидуализация стартовой АРТ /Contemporary Manage...Современное лечение ВИЧ:  индивидуализация стартовой АРТ /Contemporary Manage...
Современное лечение ВИЧ: индивидуализация стартовой АРТ /Contemporary Manage...
 
Современное лечение ВИЧ: модификация АРТ у пациентов с вирусной супрессией и ...
Современное лечение ВИЧ: модификация АРТ у пациентов с вирусной супрессией и ...Современное лечение ВИЧ: модификация АРТ у пациентов с вирусной супрессией и ...
Современное лечение ВИЧ: модификация АРТ у пациентов с вирусной супрессией и ...
 
Антиретровирусные средства и хроническая болезнь почек.Exposure to antiretrov...
Антиретровирусные средства и хроническая болезнь почек.Exposure to antiretrov...Антиретровирусные средства и хроническая болезнь почек.Exposure to antiretrov...
Антиретровирусные средства и хроническая болезнь почек.Exposure to antiretrov...
 
Современное лечение ВИЧ: новые парадигмы в АРТ / Contemporary Management of H...
Современное лечение ВИЧ: новые парадигмы в АРТ / Contemporary Management of H...Современное лечение ВИЧ: новые парадигмы в АРТ / Contemporary Management of H...
Современное лечение ВИЧ: новые парадигмы в АРТ / Contemporary Management of H...
 
Contemporary Management of HIV.How Common Comorbidities Affect ART Management...
Contemporary Management of HIV.How Common Comorbidities Affect ART Management...Contemporary Management of HIV.How Common Comorbidities Affect ART Management...
Contemporary Management of HIV.How Common Comorbidities Affect ART Management...
 
Highlights of AIDS 2016
Highlights of AIDS 2016Highlights of AIDS 2016
Highlights of AIDS 2016
 
Key Slides on Individualizing ART Management Based on Treatment Safety and To...
Key Slides on Individualizing ART Management Based on Treatment Safety and To...Key Slides on Individualizing ART Management Based on Treatment Safety and To...
Key Slides on Individualizing ART Management Based on Treatment Safety and To...
 
Сравнение режимов лечения ВИЧ в разрезе различных клинических сценариев.ART...
Сравнение  режимов лечения ВИЧ в  разрезе различных клинических сценариев.ART...Сравнение  режимов лечения ВИЧ в  разрезе различных клинических сценариев.ART...
Сравнение режимов лечения ВИЧ в разрезе различных клинических сценариев.ART...
 
Современное лечение ВИЧ : лечение возрастных пациентов.2017/Contemporary Mana...
Современное лечение ВИЧ : лечение возрастных пациентов.2017/Contemporary Mana...Современное лечение ВИЧ : лечение возрастных пациентов.2017/Contemporary Mana...
Современное лечение ВИЧ : лечение возрастных пациентов.2017/Contemporary Mana...
 
Современное лечение ВИЧ : АРТ как профилактика.Contemporary Management of HIV...
Современное лечение ВИЧ : АРТ как профилактика.Contemporary Management of HIV...Современное лечение ВИЧ : АРТ как профилактика.Contemporary Management of HIV...
Современное лечение ВИЧ : АРТ как профилактика.Contemporary Management of HIV...
 
HIV Alert:ART Considerations for Aging Patients.2018
HIV Alert:ART Considerations for Aging Patients.2018HIV Alert:ART Considerations for Aging Patients.2018
HIV Alert:ART Considerations for Aging Patients.2018
 
Современное лечение ВИЧ: лечение многократно леченных пациентов с резистентно...
Современное лечение ВИЧ: лечение многократно леченных пациентов с резистентно...Современное лечение ВИЧ: лечение многократно леченных пациентов с резистентно...
Современное лечение ВИЧ: лечение многократно леченных пациентов с резистентно...
 
Innovative Paradigms for ART.2019
Innovative Paradigms for ART.2019Innovative Paradigms for ART.2019
Innovative Paradigms for ART.2019
 
Why, when, and how to use pre exposure prophylaxis for hiv acquisition. 2014
Why, when, and how to use pre exposure prophylaxis for hiv acquisition. 2014Why, when, and how to use pre exposure prophylaxis for hiv acquisition. 2014
Why, when, and how to use pre exposure prophylaxis for hiv acquisition. 2014
 
Современное лечение ВИЧ: лечение ВИЧ у пациентов с вирусными гепатитами.Conte...
Современное лечение ВИЧ: лечение ВИЧ у пациентов с вирусными гепатитами.Conte...Современное лечение ВИЧ: лечение ВИЧ у пациентов с вирусными гепатитами.Conte...
Современное лечение ВИЧ: лечение ВИЧ у пациентов с вирусными гепатитами.Conte...
 
Determining Candidacy and Strategies for ART Modification.2019
Determining Candidacy and Strategies for ART Modification.2019Determining Candidacy and Strategies for ART Modification.2019
Determining Candidacy and Strategies for ART Modification.2019
 
АРТ в 2016-2017 гг: неизменная потребность в индивидуализации лечения для улу...
АРТ в 2016-2017 гг: неизменная потребность в индивидуализации лечения для улу...АРТ в 2016-2017 гг: неизменная потребность в индивидуализации лечения для улу...
АРТ в 2016-2017 гг: неизменная потребность в индивидуализации лечения для улу...
 
Incorporating New ART Options Into First-line and Switch Strategies for HIV C...
Incorporating New ART Options Into First-line and Switch Strategies for HIV C...Incorporating New ART Options Into First-line and Switch Strategies for HIV C...
Incorporating New ART Options Into First-line and Switch Strategies for HIV C...
 

Ähnlich wie Современное лечение ВИЧ: когда начинать, чем начинать. Contemporary Management of HIV. When to Start, What to Start.2016

IAS 2015.8th IAS Conference on HIV Pathogenesis, Treatment and Prevention
IAS 2015.8th IAS Conference on HIV Pathogenesis, Treatment and PreventionIAS 2015.8th IAS Conference on HIV Pathogenesis, Treatment and Prevention
IAS 2015.8th IAS Conference on HIV Pathogenesis, Treatment and Preventionhivlifeinfo
 
A New Options for HIV Prevention.The Role of Antiretrovirals
A New Options for HIV Prevention.The Role of AntiretroviralsA New Options for HIV Prevention.The Role of Antiretrovirals
A New Options for HIV Prevention.The Role of Antiretroviralshivlifeinfo
 
Management of HIV - Post-Exposure Prophylaxis Management of HIV - Post-Expo...
Management of HIV - Post-Exposure Prophylaxis 	 Management of HIV - Post-Expo...Management of HIV - Post-Exposure Prophylaxis 	 Management of HIV - Post-Expo...
Management of HIV - Post-Exposure Prophylaxis Management of HIV - Post-Expo...MedicineAndHealthUSA
 
Contemporary Management of HIV. New Data From IDWeek 2018 and Other Fall 2018...
Contemporary Management of HIV. New Data From IDWeek 2018 and Other Fall 2018...Contemporary Management of HIV. New Data From IDWeek 2018 and Other Fall 2018...
Contemporary Management of HIV. New Data From IDWeek 2018 and Other Fall 2018...hivlifeinfo
 
Случаи и разногласия по ВИЧ в 2019 году: европейские перспективы / Cases and...
 Случаи и разногласия по ВИЧ в 2019 году: европейские перспективы / Cases and... Случаи и разногласия по ВИЧ в 2019 году: европейские перспективы / Cases and...
Случаи и разногласия по ВИЧ в 2019 году: европейские перспективы / Cases and...hivlifeinfo
 
Highlights of AIDS 2014 .CCO Official Conference Coverage of the 20th Interna...
Highlights of AIDS 2014 .CCO Official Conference Coverage of the 20th Interna...Highlights of AIDS 2014 .CCO Official Conference Coverage of the 20th Interna...
Highlights of AIDS 2014 .CCO Official Conference Coverage of the 20th Interna...Hivlife Info
 
A new options for hiv prevention slides.2013
A new options for hiv prevention slides.2013A new options for hiv prevention slides.2013
A new options for hiv prevention slides.2013Hivlife Info
 
What I Use and Why: Expert Strategies for Selecting the Best ART Regimen for ...
What I Use and Why: Expert Strategies for Selecting the Best ART Regimen for ...What I Use and Why: Expert Strategies for Selecting the Best ART Regimen for ...
What I Use and Why: Expert Strategies for Selecting the Best ART Regimen for ...hivlifeinfo
 
Hiv eye update 2013
Hiv eye update 2013Hiv eye update 2013
Hiv eye update 2013etedaldi
 
New Data on Resistance to DAAs and Implications for Therapy.2015
New Data on Resistance to DAAs and Implications for Therapy.2015New Data on Resistance to DAAs and Implications for Therapy.2015
New Data on Resistance to DAAs and Implications for Therapy.2015hivlifeinfo
 
Contemporary Management of HIV. New Data From AIDS 2018
Contemporary Management of HIV. New Data From AIDS 2018Contemporary Management of HIV. New Data From AIDS 2018
Contemporary Management of HIV. New Data From AIDS 2018hivlifeinfo
 
Start impaact june 7 2011
Start impaact june 7 2011Start impaact june 7 2011
Start impaact june 7 2011Phil Boehmer
 
Start impaact june 7 2011
Start impaact june 7 2011Start impaact june 7 2011
Start impaact june 7 2011Phil Boehmer
 
Key Slides on ART for HIV : Evolving Concepts and Innovative Strategies.2020
Key Slides on ART for HIV : Evolving Concepts and Innovative Strategies.2020Key Slides on ART for HIV : Evolving Concepts and Innovative Strategies.2020
Key Slides on ART for HIV : Evolving Concepts and Innovative Strategies.2020hivlifeinfo
 
High Sensitivity HIV Testing and Translational Science around PrEP
High Sensitivity HIV Testing and Translational Science around PrEPHigh Sensitivity HIV Testing and Translational Science around PrEP
High Sensitivity HIV Testing and Translational Science around PrEPHopkinsCFAR
 
Cовременное лечение ВИЧ : новые данные с конференции CROI 2017/ Contemporary...
Cовременное лечение ВИЧ : новые данные с  конференции CROI 2017/ Contemporary...Cовременное лечение ВИЧ : новые данные с  конференции CROI 2017/ Contemporary...
Cовременное лечение ВИЧ : новые данные с конференции CROI 2017/ Contemporary...hivlifeinfo
 
Clinical Impact of Data From the CROI 2015,Seattle
Clinical Impact of Data From the CROI 2015,SeattleClinical Impact of Data From the CROI 2015,Seattle
Clinical Impact of Data From the CROI 2015,SeattleHivlife Info
 
Clinical Impact of Data From the CROI 2015,Seattle
Clinical Impact of Data From the CROI 2015,SeattleClinical Impact of Data From the CROI 2015,Seattle
Clinical Impact of Data From the CROI 2015,Seattlehivlifeinfo
 

Ähnlich wie Современное лечение ВИЧ: когда начинать, чем начинать. Contemporary Management of HIV. When to Start, What to Start.2016 (20)

IAS 2015.8th IAS Conference on HIV Pathogenesis, Treatment and Prevention
IAS 2015.8th IAS Conference on HIV Pathogenesis, Treatment and PreventionIAS 2015.8th IAS Conference on HIV Pathogenesis, Treatment and Prevention
IAS 2015.8th IAS Conference on HIV Pathogenesis, Treatment and Prevention
 
A New Options for HIV Prevention.The Role of Antiretrovirals
A New Options for HIV Prevention.The Role of AntiretroviralsA New Options for HIV Prevention.The Role of Antiretrovirals
A New Options for HIV Prevention.The Role of Antiretrovirals
 
Top Ten HIV Clinical Controversies 2014
Top Ten HIV Clinical Controversies 2014Top Ten HIV Clinical Controversies 2014
Top Ten HIV Clinical Controversies 2014
 
Management of HIV - Post-Exposure Prophylaxis Management of HIV - Post-Expo...
Management of HIV - Post-Exposure Prophylaxis 	 Management of HIV - Post-Expo...Management of HIV - Post-Exposure Prophylaxis 	 Management of HIV - Post-Expo...
Management of HIV - Post-Exposure Prophylaxis Management of HIV - Post-Expo...
 
Contemporary Management of HIV. New Data From IDWeek 2018 and Other Fall 2018...
Contemporary Management of HIV. New Data From IDWeek 2018 and Other Fall 2018...Contemporary Management of HIV. New Data From IDWeek 2018 and Other Fall 2018...
Contemporary Management of HIV. New Data From IDWeek 2018 and Other Fall 2018...
 
Случаи и разногласия по ВИЧ в 2019 году: европейские перспективы / Cases and...
 Случаи и разногласия по ВИЧ в 2019 году: европейские перспективы / Cases and... Случаи и разногласия по ВИЧ в 2019 году: европейские перспективы / Cases and...
Случаи и разногласия по ВИЧ в 2019 году: европейские перспективы / Cases and...
 
Highlights of AIDS 2014 .CCO Official Conference Coverage of the 20th Interna...
Highlights of AIDS 2014 .CCO Official Conference Coverage of the 20th Interna...Highlights of AIDS 2014 .CCO Official Conference Coverage of the 20th Interna...
Highlights of AIDS 2014 .CCO Official Conference Coverage of the 20th Interna...
 
A new options for hiv prevention slides.2013
A new options for hiv prevention slides.2013A new options for hiv prevention slides.2013
A new options for hiv prevention slides.2013
 
What I Use and Why: Expert Strategies for Selecting the Best ART Regimen for ...
What I Use and Why: Expert Strategies for Selecting the Best ART Regimen for ...What I Use and Why: Expert Strategies for Selecting the Best ART Regimen for ...
What I Use and Why: Expert Strategies for Selecting the Best ART Regimen for ...
 
Hiv eye update 2013
Hiv eye update 2013Hiv eye update 2013
Hiv eye update 2013
 
Cco retroviruses_2013_art_slides
Cco  retroviruses_2013_art_slidesCco  retroviruses_2013_art_slides
Cco retroviruses_2013_art_slides
 
New Data on Resistance to DAAs and Implications for Therapy.2015
New Data on Resistance to DAAs and Implications for Therapy.2015New Data on Resistance to DAAs and Implications for Therapy.2015
New Data on Resistance to DAAs and Implications for Therapy.2015
 
Contemporary Management of HIV. New Data From AIDS 2018
Contemporary Management of HIV. New Data From AIDS 2018Contemporary Management of HIV. New Data From AIDS 2018
Contemporary Management of HIV. New Data From AIDS 2018
 
Start impaact june 7 2011
Start impaact june 7 2011Start impaact june 7 2011
Start impaact june 7 2011
 
Start impaact june 7 2011
Start impaact june 7 2011Start impaact june 7 2011
Start impaact june 7 2011
 
Key Slides on ART for HIV : Evolving Concepts and Innovative Strategies.2020
Key Slides on ART for HIV : Evolving Concepts and Innovative Strategies.2020Key Slides on ART for HIV : Evolving Concepts and Innovative Strategies.2020
Key Slides on ART for HIV : Evolving Concepts and Innovative Strategies.2020
 
High Sensitivity HIV Testing and Translational Science around PrEP
High Sensitivity HIV Testing and Translational Science around PrEPHigh Sensitivity HIV Testing and Translational Science around PrEP
High Sensitivity HIV Testing and Translational Science around PrEP
 
Cовременное лечение ВИЧ : новые данные с конференции CROI 2017/ Contemporary...
Cовременное лечение ВИЧ : новые данные с  конференции CROI 2017/ Contemporary...Cовременное лечение ВИЧ : новые данные с  конференции CROI 2017/ Contemporary...
Cовременное лечение ВИЧ : новые данные с конференции CROI 2017/ Contemporary...
 
Clinical Impact of Data From the CROI 2015,Seattle
Clinical Impact of Data From the CROI 2015,SeattleClinical Impact of Data From the CROI 2015,Seattle
Clinical Impact of Data From the CROI 2015,Seattle
 
Clinical Impact of Data From the CROI 2015,Seattle
Clinical Impact of Data From the CROI 2015,SeattleClinical Impact of Data From the CROI 2015,Seattle
Clinical Impact of Data From the CROI 2015,Seattle
 

Mehr von hivlifeinfo

Дискуссии о здоровом старении с ВИЧ /Key Slides on Healthy Aging With HIV.2022
Дискуссии о здоровом старении с ВИЧ /Key Slides on Healthy Aging With HIV.2022Дискуссии о здоровом старении с ВИЧ /Key Slides on Healthy Aging With HIV.2022
Дискуссии о здоровом старении с ВИЧ /Key Slides on Healthy Aging With HIV.2022hivlifeinfo
 
Основы ведения АРТ у многократно леченных пациентов 2022 / Foundations of ART...
Основы ведения АРТ у многократно леченных пациентов 2022 / Foundations of ART...Основы ведения АРТ у многократно леченных пациентов 2022 / Foundations of ART...
Основы ведения АРТ у многократно леченных пациентов 2022 / Foundations of ART...hivlifeinfo
 
Современное лечение и профилактика ВИЧ : передовые стратегии лечения у пациен...
Современное лечение и профилактика ВИЧ : передовые стратегии лечения у пациен...Современное лечение и профилактика ВИЧ : передовые стратегии лечения у пациен...
Современное лечение и профилактика ВИЧ : передовые стратегии лечения у пациен...hivlifeinfo
 
Ключевые слайды по индивидуальному выбору АРТ / Key Slides on Individualized ...
Ключевые слайды по индивидуальному выбору АРТ / Key Slides on Individualized ...Ключевые слайды по индивидуальному выбору АРТ / Key Slides on Individualized ...
Ключевые слайды по индивидуальному выбору АРТ / Key Slides on Individualized ...hivlifeinfo
 
Ключевые решения в лечении ВИЧ: оптимизация стратегии лечения для пациентов с...
Ключевые решения в лечении ВИЧ: оптимизация стратегии лечения для пациентов с...Ключевые решения в лечении ВИЧ: оптимизация стратегии лечения для пациентов с...
Ключевые решения в лечении ВИЧ: оптимизация стратегии лечения для пациентов с...hivlifeinfo
 
Современное лечение ВИЧ: модификация АРТ у пациентов с вирусологической супре...
Современное лечение ВИЧ: модификация АРТ у пациентов с вирусологической супре...Современное лечение ВИЧ: модификация АРТ у пациентов с вирусологической супре...
Современное лечение ВИЧ: модификация АРТ у пациентов с вирусологической супре...hivlifeinfo
 
Clinical Impact of New Data From AIDS 2020
Clinical Impact of New Data From AIDS 2020Clinical Impact of New Data From AIDS 2020
Clinical Impact of New Data From AIDS 2020hivlifeinfo
 
Слайдсет о новом в лечении ВИЧ.Key Slides on What’s Hot in HIV Treatment.2020
Слайдсет о новом в лечении ВИЧ.Key Slides on What’s Hot in HIV Treatment.2020 Слайдсет о новом в лечении ВИЧ.Key Slides on What’s Hot in HIV Treatment.2020
Слайдсет о новом в лечении ВИЧ.Key Slides on What’s Hot in HIV Treatment.2020 hivlifeinfo
 
Гиперлипопротеидемия(а) как опасное генетически обусловленное нарушение липид...
Гиперлипопротеидемия(а) как опасное генетически обусловленное нарушение липид...Гиперлипопротеидемия(а) как опасное генетически обусловленное нарушение липид...
Гиперлипопротеидемия(а) как опасное генетически обусловленное нарушение липид...hivlifeinfo
 
Физическая активность и физические тренировки как метод профилактики сердечно...
Физическая активность и физические тренировки как метод профилактики сердечно...Физическая активность и физические тренировки как метод профилактики сердечно...
Физическая активность и физические тренировки как метод профилактики сердечно...hivlifeinfo
 
Общие принципы ведения пациентов с ХБП
Общие принципы ведения пациентов с ХБПОбщие принципы ведения пациентов с ХБП
Общие принципы ведения пациентов с ХБПhivlifeinfo
 
Симптомы заболеваний почек (краткий клинический анализ)
Симптомы заболеваний почек (краткий клинический анализ)Симптомы заболеваний почек (краткий клинический анализ)
Симптомы заболеваний почек (краткий клинический анализ)hivlifeinfo
 
Клинические рекомендации «Алгоритмы специализированной медицинской помощи бол...
Клинические рекомендации «Алгоритмы специализированной медицинской помощи бол...Клинические рекомендации «Алгоритмы специализированной медицинской помощи бол...
Клинические рекомендации «Алгоритмы специализированной медицинской помощи бол...hivlifeinfo
 
Современное лечение ВИЧ.Обобщённые данные с конференции CROI 2020 / Contempor...
Современное лечение ВИЧ.Обобщённые данные с конференции CROI 2020 / Contempor...Современное лечение ВИЧ.Обобщённые данные с конференции CROI 2020 / Contempor...
Современное лечение ВИЧ.Обобщённые данные с конференции CROI 2020 / Contempor...hivlifeinfo
 
Свобода интернета 2018: делегирование репрессий.Доклад Международной Агоры
Свобода интернета 2018: делегирование репрессий.Доклад Международной АгорыСвобода интернета 2018: делегирование репрессий.Доклад Международной Агоры
Свобода интернета 2018: делегирование репрессий.Доклад Международной Агорыhivlifeinfo
 
Confronting the Challenges of HIV Care in an Aging Population.2019
Confronting the Challenges of HIV Care in an Aging Population.2019Confronting the Challenges of HIV Care in an Aging Population.2019
Confronting the Challenges of HIV Care in an Aging Population.2019hivlifeinfo
 
Современное лечение ВИЧ.Объединенные данные с конференции IAS 2019 / Contemp...
Современное лечение ВИЧ.Объединенные данные с конференции  IAS 2019 / Contemp...Современное лечение ВИЧ.Объединенные данные с конференции  IAS 2019 / Contemp...
Современное лечение ВИЧ.Объединенные данные с конференции IAS 2019 / Contemp...hivlifeinfo
 
Clinical Impact of New Data From IAS 2019
Clinical Impact of New Data From IAS 2019Clinical Impact of New Data From IAS 2019
Clinical Impact of New Data From IAS 2019hivlifeinfo
 
Предиабет-определение, риски, подходы к диагностике и профилактике сахарного ...
Предиабет-определение, риски, подходы к диагностике и профилактике сахарного ...Предиабет-определение, риски, подходы к диагностике и профилактике сахарного ...
Предиабет-определение, риски, подходы к диагностике и профилактике сахарного ...hivlifeinfo
 
Tsepamo: DTG Exposure at Conception Associated With Smaller Increase in Incid...
Tsepamo: DTG Exposure at Conception Associated With Smaller Increase in Incid...Tsepamo: DTG Exposure at Conception Associated With Smaller Increase in Incid...
Tsepamo: DTG Exposure at Conception Associated With Smaller Increase in Incid...hivlifeinfo
 

Mehr von hivlifeinfo (20)

Дискуссии о здоровом старении с ВИЧ /Key Slides on Healthy Aging With HIV.2022
Дискуссии о здоровом старении с ВИЧ /Key Slides on Healthy Aging With HIV.2022Дискуссии о здоровом старении с ВИЧ /Key Slides on Healthy Aging With HIV.2022
Дискуссии о здоровом старении с ВИЧ /Key Slides on Healthy Aging With HIV.2022
 
Основы ведения АРТ у многократно леченных пациентов 2022 / Foundations of ART...
Основы ведения АРТ у многократно леченных пациентов 2022 / Foundations of ART...Основы ведения АРТ у многократно леченных пациентов 2022 / Foundations of ART...
Основы ведения АРТ у многократно леченных пациентов 2022 / Foundations of ART...
 
Современное лечение и профилактика ВИЧ : передовые стратегии лечения у пациен...
Современное лечение и профилактика ВИЧ : передовые стратегии лечения у пациен...Современное лечение и профилактика ВИЧ : передовые стратегии лечения у пациен...
Современное лечение и профилактика ВИЧ : передовые стратегии лечения у пациен...
 
Ключевые слайды по индивидуальному выбору АРТ / Key Slides on Individualized ...
Ключевые слайды по индивидуальному выбору АРТ / Key Slides on Individualized ...Ключевые слайды по индивидуальному выбору АРТ / Key Slides on Individualized ...
Ключевые слайды по индивидуальному выбору АРТ / Key Slides on Individualized ...
 
Ключевые решения в лечении ВИЧ: оптимизация стратегии лечения для пациентов с...
Ключевые решения в лечении ВИЧ: оптимизация стратегии лечения для пациентов с...Ключевые решения в лечении ВИЧ: оптимизация стратегии лечения для пациентов с...
Ключевые решения в лечении ВИЧ: оптимизация стратегии лечения для пациентов с...
 
Современное лечение ВИЧ: модификация АРТ у пациентов с вирусологической супре...
Современное лечение ВИЧ: модификация АРТ у пациентов с вирусологической супре...Современное лечение ВИЧ: модификация АРТ у пациентов с вирусологической супре...
Современное лечение ВИЧ: модификация АРТ у пациентов с вирусологической супре...
 
Clinical Impact of New Data From AIDS 2020
Clinical Impact of New Data From AIDS 2020Clinical Impact of New Data From AIDS 2020
Clinical Impact of New Data From AIDS 2020
 
Слайдсет о новом в лечении ВИЧ.Key Slides on What’s Hot in HIV Treatment.2020
Слайдсет о новом в лечении ВИЧ.Key Slides on What’s Hot in HIV Treatment.2020 Слайдсет о новом в лечении ВИЧ.Key Slides on What’s Hot in HIV Treatment.2020
Слайдсет о новом в лечении ВИЧ.Key Slides on What’s Hot in HIV Treatment.2020
 
Гиперлипопротеидемия(а) как опасное генетически обусловленное нарушение липид...
Гиперлипопротеидемия(а) как опасное генетически обусловленное нарушение липид...Гиперлипопротеидемия(а) как опасное генетически обусловленное нарушение липид...
Гиперлипопротеидемия(а) как опасное генетически обусловленное нарушение липид...
 
Физическая активность и физические тренировки как метод профилактики сердечно...
Физическая активность и физические тренировки как метод профилактики сердечно...Физическая активность и физические тренировки как метод профилактики сердечно...
Физическая активность и физические тренировки как метод профилактики сердечно...
 
Общие принципы ведения пациентов с ХБП
Общие принципы ведения пациентов с ХБПОбщие принципы ведения пациентов с ХБП
Общие принципы ведения пациентов с ХБП
 
Симптомы заболеваний почек (краткий клинический анализ)
Симптомы заболеваний почек (краткий клинический анализ)Симптомы заболеваний почек (краткий клинический анализ)
Симптомы заболеваний почек (краткий клинический анализ)
 
Клинические рекомендации «Алгоритмы специализированной медицинской помощи бол...
Клинические рекомендации «Алгоритмы специализированной медицинской помощи бол...Клинические рекомендации «Алгоритмы специализированной медицинской помощи бол...
Клинические рекомендации «Алгоритмы специализированной медицинской помощи бол...
 
Современное лечение ВИЧ.Обобщённые данные с конференции CROI 2020 / Contempor...
Современное лечение ВИЧ.Обобщённые данные с конференции CROI 2020 / Contempor...Современное лечение ВИЧ.Обобщённые данные с конференции CROI 2020 / Contempor...
Современное лечение ВИЧ.Обобщённые данные с конференции CROI 2020 / Contempor...
 
Свобода интернета 2018: делегирование репрессий.Доклад Международной Агоры
Свобода интернета 2018: делегирование репрессий.Доклад Международной АгорыСвобода интернета 2018: делегирование репрессий.Доклад Международной Агоры
Свобода интернета 2018: делегирование репрессий.Доклад Международной Агоры
 
Confronting the Challenges of HIV Care in an Aging Population.2019
Confronting the Challenges of HIV Care in an Aging Population.2019Confronting the Challenges of HIV Care in an Aging Population.2019
Confronting the Challenges of HIV Care in an Aging Population.2019
 
Современное лечение ВИЧ.Объединенные данные с конференции IAS 2019 / Contemp...
Современное лечение ВИЧ.Объединенные данные с конференции  IAS 2019 / Contemp...Современное лечение ВИЧ.Объединенные данные с конференции  IAS 2019 / Contemp...
Современное лечение ВИЧ.Объединенные данные с конференции IAS 2019 / Contemp...
 
Clinical Impact of New Data From IAS 2019
Clinical Impact of New Data From IAS 2019Clinical Impact of New Data From IAS 2019
Clinical Impact of New Data From IAS 2019
 
Предиабет-определение, риски, подходы к диагностике и профилактике сахарного ...
Предиабет-определение, риски, подходы к диагностике и профилактике сахарного ...Предиабет-определение, риски, подходы к диагностике и профилактике сахарного ...
Предиабет-определение, риски, подходы к диагностике и профилактике сахарного ...
 
Tsepamo: DTG Exposure at Conception Associated With Smaller Increase in Incid...
Tsepamo: DTG Exposure at Conception Associated With Smaller Increase in Incid...Tsepamo: DTG Exposure at Conception Associated With Smaller Increase in Incid...
Tsepamo: DTG Exposure at Conception Associated With Smaller Increase in Incid...
 

Kürzlich hochgeladen

Call Girls Service Chennai Jiya 7001305949 Independent Escort Service Chennai
Call Girls Service Chennai Jiya 7001305949 Independent Escort Service ChennaiCall Girls Service Chennai Jiya 7001305949 Independent Escort Service Chennai
Call Girls Service Chennai Jiya 7001305949 Independent Escort Service ChennaiNehru place Escorts
 
Russian Call Girls Chickpet - 7001305949 Booking and charges genuine rate for...
Russian Call Girls Chickpet - 7001305949 Booking and charges genuine rate for...Russian Call Girls Chickpet - 7001305949 Booking and charges genuine rate for...
Russian Call Girls Chickpet - 7001305949 Booking and charges genuine rate for...narwatsonia7
 
Call Girls Viman Nagar 7001305949 All Area Service COD available Any Time
Call Girls Viman Nagar 7001305949 All Area Service COD available Any TimeCall Girls Viman Nagar 7001305949 All Area Service COD available Any Time
Call Girls Viman Nagar 7001305949 All Area Service COD available Any Timevijaych2041
 
Call Girls Kanakapura Road Just Call 7001305949 Top Class Call Girl Service A...
Call Girls Kanakapura Road Just Call 7001305949 Top Class Call Girl Service A...Call Girls Kanakapura Road Just Call 7001305949 Top Class Call Girl Service A...
Call Girls Kanakapura Road Just Call 7001305949 Top Class Call Girl Service A...narwatsonia7
 
Call Girl Koramangala | 7001305949 At Low Cost Cash Payment Booking
Call Girl Koramangala | 7001305949 At Low Cost Cash Payment BookingCall Girl Koramangala | 7001305949 At Low Cost Cash Payment Booking
Call Girl Koramangala | 7001305949 At Low Cost Cash Payment Bookingnarwatsonia7
 
Call Girls Electronic City Just Call 7001305949 Top Class Call Girl Service A...
Call Girls Electronic City Just Call 7001305949 Top Class Call Girl Service A...Call Girls Electronic City Just Call 7001305949 Top Class Call Girl Service A...
Call Girls Electronic City Just Call 7001305949 Top Class Call Girl Service A...narwatsonia7
 
VIP Call Girls Mumbai Arpita 9910780858 Independent Escort Service Mumbai
VIP Call Girls Mumbai Arpita 9910780858 Independent Escort Service MumbaiVIP Call Girls Mumbai Arpita 9910780858 Independent Escort Service Mumbai
VIP Call Girls Mumbai Arpita 9910780858 Independent Escort Service Mumbaisonalikaur4
 
Call Girls Jayanagar Just Call 7001305949 Top Class Call Girl Service Available
Call Girls Jayanagar Just Call 7001305949 Top Class Call Girl Service AvailableCall Girls Jayanagar Just Call 7001305949 Top Class Call Girl Service Available
Call Girls Jayanagar Just Call 7001305949 Top Class Call Girl Service Availablenarwatsonia7
 
Call Girls Frazer Town Just Call 7001305949 Top Class Call Girl Service Avail...
Call Girls Frazer Town Just Call 7001305949 Top Class Call Girl Service Avail...Call Girls Frazer Town Just Call 7001305949 Top Class Call Girl Service Avail...
Call Girls Frazer Town Just Call 7001305949 Top Class Call Girl Service Avail...narwatsonia7
 
Book Call Girls in Kasavanahalli - 7001305949 with real photos and phone numbers
Book Call Girls in Kasavanahalli - 7001305949 with real photos and phone numbersBook Call Girls in Kasavanahalli - 7001305949 with real photos and phone numbers
Book Call Girls in Kasavanahalli - 7001305949 with real photos and phone numbersnarwatsonia7
 
Housewife Call Girls Hsr Layout - Call 7001305949 Rs-3500 with A/C Room Cash ...
Housewife Call Girls Hsr Layout - Call 7001305949 Rs-3500 with A/C Room Cash ...Housewife Call Girls Hsr Layout - Call 7001305949 Rs-3500 with A/C Room Cash ...
Housewife Call Girls Hsr Layout - Call 7001305949 Rs-3500 with A/C Room Cash ...narwatsonia7
 
Call Girl Service Bidadi - For 7001305949 Cheap & Best with original Photos
Call Girl Service Bidadi - For 7001305949 Cheap & Best with original PhotosCall Girl Service Bidadi - For 7001305949 Cheap & Best with original Photos
Call Girl Service Bidadi - For 7001305949 Cheap & Best with original Photosnarwatsonia7
 
Noida Sector 135 Call Girls ( 9873940964 ) Book Hot And Sexy Girls In A Few C...
Noida Sector 135 Call Girls ( 9873940964 ) Book Hot And Sexy Girls In A Few C...Noida Sector 135 Call Girls ( 9873940964 ) Book Hot And Sexy Girls In A Few C...
Noida Sector 135 Call Girls ( 9873940964 ) Book Hot And Sexy Girls In A Few C...rajnisinghkjn
 
High Profile Call Girls Jaipur Vani 8445551418 Independent Escort Service Jaipur
High Profile Call Girls Jaipur Vani 8445551418 Independent Escort Service JaipurHigh Profile Call Girls Jaipur Vani 8445551418 Independent Escort Service Jaipur
High Profile Call Girls Jaipur Vani 8445551418 Independent Escort Service Jaipurparulsinha
 
Call Girl Lucknow Mallika 7001305949 Independent Escort Service Lucknow
Call Girl Lucknow Mallika 7001305949 Independent Escort Service LucknowCall Girl Lucknow Mallika 7001305949 Independent Escort Service Lucknow
Call Girl Lucknow Mallika 7001305949 Independent Escort Service Lucknownarwatsonia7
 
Call Girls Service in Bommanahalli - 7001305949 with real photos and phone nu...
Call Girls Service in Bommanahalli - 7001305949 with real photos and phone nu...Call Girls Service in Bommanahalli - 7001305949 with real photos and phone nu...
Call Girls Service in Bommanahalli - 7001305949 with real photos and phone nu...narwatsonia7
 
Hematology and Immunology - Leukocytes Functions
Hematology and Immunology - Leukocytes FunctionsHematology and Immunology - Leukocytes Functions
Hematology and Immunology - Leukocytes FunctionsMedicoseAcademics
 
Russian Call Girls Gunjur Mugalur Road : 7001305949 High Profile Model Escort...
Russian Call Girls Gunjur Mugalur Road : 7001305949 High Profile Model Escort...Russian Call Girls Gunjur Mugalur Road : 7001305949 High Profile Model Escort...
Russian Call Girls Gunjur Mugalur Road : 7001305949 High Profile Model Escort...narwatsonia7
 
Call Girls Whitefield Just Call 7001305949 Top Class Call Girl Service Available
Call Girls Whitefield Just Call 7001305949 Top Class Call Girl Service AvailableCall Girls Whitefield Just Call 7001305949 Top Class Call Girl Service Available
Call Girls Whitefield Just Call 7001305949 Top Class Call Girl Service Availablenarwatsonia7
 
Call Girls Jp Nagar Just Call 7001305949 Top Class Call Girl Service Available
Call Girls Jp Nagar Just Call 7001305949 Top Class Call Girl Service AvailableCall Girls Jp Nagar Just Call 7001305949 Top Class Call Girl Service Available
Call Girls Jp Nagar Just Call 7001305949 Top Class Call Girl Service Availablenarwatsonia7
 

Kürzlich hochgeladen (20)

Call Girls Service Chennai Jiya 7001305949 Independent Escort Service Chennai
Call Girls Service Chennai Jiya 7001305949 Independent Escort Service ChennaiCall Girls Service Chennai Jiya 7001305949 Independent Escort Service Chennai
Call Girls Service Chennai Jiya 7001305949 Independent Escort Service Chennai
 
Russian Call Girls Chickpet - 7001305949 Booking and charges genuine rate for...
Russian Call Girls Chickpet - 7001305949 Booking and charges genuine rate for...Russian Call Girls Chickpet - 7001305949 Booking and charges genuine rate for...
Russian Call Girls Chickpet - 7001305949 Booking and charges genuine rate for...
 
Call Girls Viman Nagar 7001305949 All Area Service COD available Any Time
Call Girls Viman Nagar 7001305949 All Area Service COD available Any TimeCall Girls Viman Nagar 7001305949 All Area Service COD available Any Time
Call Girls Viman Nagar 7001305949 All Area Service COD available Any Time
 
Call Girls Kanakapura Road Just Call 7001305949 Top Class Call Girl Service A...
Call Girls Kanakapura Road Just Call 7001305949 Top Class Call Girl Service A...Call Girls Kanakapura Road Just Call 7001305949 Top Class Call Girl Service A...
Call Girls Kanakapura Road Just Call 7001305949 Top Class Call Girl Service A...
 
Call Girl Koramangala | 7001305949 At Low Cost Cash Payment Booking
Call Girl Koramangala | 7001305949 At Low Cost Cash Payment BookingCall Girl Koramangala | 7001305949 At Low Cost Cash Payment Booking
Call Girl Koramangala | 7001305949 At Low Cost Cash Payment Booking
 
Call Girls Electronic City Just Call 7001305949 Top Class Call Girl Service A...
Call Girls Electronic City Just Call 7001305949 Top Class Call Girl Service A...Call Girls Electronic City Just Call 7001305949 Top Class Call Girl Service A...
Call Girls Electronic City Just Call 7001305949 Top Class Call Girl Service A...
 
VIP Call Girls Mumbai Arpita 9910780858 Independent Escort Service Mumbai
VIP Call Girls Mumbai Arpita 9910780858 Independent Escort Service MumbaiVIP Call Girls Mumbai Arpita 9910780858 Independent Escort Service Mumbai
VIP Call Girls Mumbai Arpita 9910780858 Independent Escort Service Mumbai
 
Call Girls Jayanagar Just Call 7001305949 Top Class Call Girl Service Available
Call Girls Jayanagar Just Call 7001305949 Top Class Call Girl Service AvailableCall Girls Jayanagar Just Call 7001305949 Top Class Call Girl Service Available
Call Girls Jayanagar Just Call 7001305949 Top Class Call Girl Service Available
 
Call Girls Frazer Town Just Call 7001305949 Top Class Call Girl Service Avail...
Call Girls Frazer Town Just Call 7001305949 Top Class Call Girl Service Avail...Call Girls Frazer Town Just Call 7001305949 Top Class Call Girl Service Avail...
Call Girls Frazer Town Just Call 7001305949 Top Class Call Girl Service Avail...
 
Book Call Girls in Kasavanahalli - 7001305949 with real photos and phone numbers
Book Call Girls in Kasavanahalli - 7001305949 with real photos and phone numbersBook Call Girls in Kasavanahalli - 7001305949 with real photos and phone numbers
Book Call Girls in Kasavanahalli - 7001305949 with real photos and phone numbers
 
Housewife Call Girls Hsr Layout - Call 7001305949 Rs-3500 with A/C Room Cash ...
Housewife Call Girls Hsr Layout - Call 7001305949 Rs-3500 with A/C Room Cash ...Housewife Call Girls Hsr Layout - Call 7001305949 Rs-3500 with A/C Room Cash ...
Housewife Call Girls Hsr Layout - Call 7001305949 Rs-3500 with A/C Room Cash ...
 
Call Girl Service Bidadi - For 7001305949 Cheap & Best with original Photos
Call Girl Service Bidadi - For 7001305949 Cheap & Best with original PhotosCall Girl Service Bidadi - For 7001305949 Cheap & Best with original Photos
Call Girl Service Bidadi - For 7001305949 Cheap & Best with original Photos
 
Noida Sector 135 Call Girls ( 9873940964 ) Book Hot And Sexy Girls In A Few C...
Noida Sector 135 Call Girls ( 9873940964 ) Book Hot And Sexy Girls In A Few C...Noida Sector 135 Call Girls ( 9873940964 ) Book Hot And Sexy Girls In A Few C...
Noida Sector 135 Call Girls ( 9873940964 ) Book Hot And Sexy Girls In A Few C...
 
High Profile Call Girls Jaipur Vani 8445551418 Independent Escort Service Jaipur
High Profile Call Girls Jaipur Vani 8445551418 Independent Escort Service JaipurHigh Profile Call Girls Jaipur Vani 8445551418 Independent Escort Service Jaipur
High Profile Call Girls Jaipur Vani 8445551418 Independent Escort Service Jaipur
 
Call Girl Lucknow Mallika 7001305949 Independent Escort Service Lucknow
Call Girl Lucknow Mallika 7001305949 Independent Escort Service LucknowCall Girl Lucknow Mallika 7001305949 Independent Escort Service Lucknow
Call Girl Lucknow Mallika 7001305949 Independent Escort Service Lucknow
 
Call Girls Service in Bommanahalli - 7001305949 with real photos and phone nu...
Call Girls Service in Bommanahalli - 7001305949 with real photos and phone nu...Call Girls Service in Bommanahalli - 7001305949 with real photos and phone nu...
Call Girls Service in Bommanahalli - 7001305949 with real photos and phone nu...
 
Hematology and Immunology - Leukocytes Functions
Hematology and Immunology - Leukocytes FunctionsHematology and Immunology - Leukocytes Functions
Hematology and Immunology - Leukocytes Functions
 
Russian Call Girls Gunjur Mugalur Road : 7001305949 High Profile Model Escort...
Russian Call Girls Gunjur Mugalur Road : 7001305949 High Profile Model Escort...Russian Call Girls Gunjur Mugalur Road : 7001305949 High Profile Model Escort...
Russian Call Girls Gunjur Mugalur Road : 7001305949 High Profile Model Escort...
 
Call Girls Whitefield Just Call 7001305949 Top Class Call Girl Service Available
Call Girls Whitefield Just Call 7001305949 Top Class Call Girl Service AvailableCall Girls Whitefield Just Call 7001305949 Top Class Call Girl Service Available
Call Girls Whitefield Just Call 7001305949 Top Class Call Girl Service Available
 
Call Girls Jp Nagar Just Call 7001305949 Top Class Call Girl Service Available
Call Girls Jp Nagar Just Call 7001305949 Top Class Call Girl Service AvailableCall Girls Jp Nagar Just Call 7001305949 Top Class Call Girl Service Available
Call Girls Jp Nagar Just Call 7001305949 Top Class Call Girl Service Available
 

Современное лечение ВИЧ: когда начинать, чем начинать. Contemporary Management of HIV. When to Start, What to Start.2016

  • 1. Contemporary Management of HIV: When to Start, What to Start This program is supported by an independent educational grant from ViiV Healthcare
  • 2. Slide credit: clinicaloptions.com About These Slides  Please feel free to use, update, and share some or all of these slides in your noncommercial presentations to colleagues or patients  When using our slides, please retain the source attribution:  These slides may not be published, posted online, or used in commercial presentations without permission. Please contact permissions@clinicaloptions.com for details
  • 3. Program Director and Core Faculty Program Chair Eric S. Daar, MD Chief, Division of HIV Medicine Harbor-UCLA Medical Center Professor of Medicine David Geffen School of Medicine at UCLA Los Angeles, California Daniel R. Kuritzkes, MD Chief, Division of Infectious Diseases Brigham and Women’s Hospital Professor of Medicine Harvard Medical School Boston, Massachusetts
  • 4. Faculty Disclosure Information Eric S. Daar, MD, has disclosed that he has received consulting fees from AbbVie, Bristol-Myers Squibb, Gilead Sciences, Janssen, Merck, Teva, and ViiV and funds for research support from Bristol-Myers Squibb, Gilead Sciences, Merck, and ViiV. Daniel R. Kuritzkes, MD, has disclosed that he has received consulting fees from Gilead Sciences, Merck, and ViiV; fees for non-CME/CE services received directly from a commercial interest or their agents (eg, speaker bureaus) from Gilead Sciences and Merck; and funds for research support from Gilead Sciences and ViiV.
  • 5. Case 1: Patient Newly Diagnosed With HIV Infection
  • 6. Case 1: 22-Yr-Old Man Recently Diagnosed With HIV  22-yr-old black man presents with newly diagnosed HIV infection, after testing positive on routine screening  Tested HIV seronegative 6 mos ago  No significant medical problems, no medication use  Initial laboratory studies, including lipids, renal, and hepatic function are normal  CD4+ cell count 726 cells/mm3 (40%), HIV-1 RNA 2130 copies/mL  HIV genotype WT, HBV immune, HCV negative, HLA- B*5701 negative  He is unsure if he should start antiretroviral therapy
  • 7. Case 1: Current Presentation  Pt expresses concern about having to take medicines every day and is worried about adverse events  He is a nonsmoker and has no family history of heart disease  He has multiple sex partners, but says he “almost always” uses condoms when he has sex
  • 8. Would you recommend antiretroviral therapy for this pt? A. Yes B. No C. Unsure  22-yr-old black man with recent HIV seroconversion  HIV-1 RNA 2130 copies/mL, CD4+ cell count 726 cells/mm3  No medical problems  Labs normal, HIV GT WT, HBV immune, HCV negative, HLA-B*5701 negative  Multiple sex partners
  • 9. START: Immediate vs Deferred Therapy for Asymptomatic, ART-Naive Pts Immediate ART ART initiated immediately following randomization (n = 2326) INSIGHT START Study Group. N Engl J Med. 2015;373:795-807. Lundgren J, et al. IAS 2015. Abstract MOSY0302. Deferred ART Deferred until CD4+ cell count ≤ 350 cells/mm3 , AIDS, or event requiring ART (n = 2359) HIV-positive, ART-naive adults with CD4+ cell count > 500 cells/mm3 (N = 4685) Study closed by DSMB following interim analysis Slide credit: clinicaloptions.com
  • 10. START: Primary Outcome Primary Endpoint Immediate ART Deferred ART No. with event (%) 42 (1.8) 96 (4.1) Rate/100 PY 0.60 1.38 HR (immediate/deferred) 0.43 (95% CI: 0.30-0.62; P < .001)  57% reduced risk of serious events or death with immediate ART  68% of primary endpoints occurred in pts with CD4+ cell counts > 500 cells/mm3 10 8 6 4 2 0 CumulativePercent WithEvent 0 6 12 18 24 30 36 42 48 54 60 Mos INSIGHT START Group. N Engl J Med. 2015;373:795-807. Lundgren J, et al. IAS 2015. Abstract MOSY0302. 2.5 5.3 Immediate ART Deferred ART Slide credit: clinicaloptions.com
  • 11. START: Serious AIDS Events  72% reduced risk of serious AIDS events with immediate ART INSIGHT START Study Group. N Engl J Med. 2015;373:795-807. Lundgren J, et al. IAS 2015. Abstract MOSY0302. Serious AIDS Events Immediate ART Deferred ART No. with event (%) 14 50 Rate/100 PY 0.20 0.72 HR (immediate/deferred) 0.28 (95% CI: 0.15-0.50; P < .001) 0 6 12 18 24 30 36 42 48 54 60 Mos 10 8 6 4 2 0 CumulativePercent WithanEvent Immediate ART Deferred ART Slide credit: clinicaloptions.com
  • 12. START: Serious Non-AIDS Events  39% reduced risk of serious non-AIDS events with immediate ART 0 6 12 18 24 30 36 42 48 54 60 Mos 10 8 6 4 2 0 CumulativePercent WithanEvent Serious Non-AIDS Events Immediate ART Deferred ART No. with event (%) 29 47 Rate/100 PY 0.42 0.67 HR (immediate/deferred) 0.61 (95% CI: 0.38-0.97; P = .04) Immediate ART Deferred ART Slide credit: clinicaloptions.com INSIGHT START Study Group. N Engl J Med. 2015;373:795-807. Lundgren J, et al. IAS 2015. Abstract MOSY0302.
  • 13. START: Reduced Risk of Cancers With Immediate ART *Immediate ART: squamous cell carcinoma, plasma cell myeloma, bladder cancer, fibrosarcoma. Deferred ART: gastric adenocarcinoma, breast cancer, ureteric cancer, malignant melanoma, myeloid leukemia, thyroid cancer, leiomyosarcoma, liver cancer, squamous cell carcinoma of head and neck. Cancer Event Immediate ART Deferred ART Kaposi’s sarcoma 1 11 Lymphoma, NHL + HL 3 10 Prostate cancer 2 3 Lung cancer 2 2 Anal cancer 1 2 Cervical or testis cancer 1 2 Other types* 4 9 Total 14 39 INSIGHT START Study Group. N Engl J Med. 2015;373:795-807. Lundgren J, et al. IAS 2015. Abstract MOSY0302. Slide credit: clinicaloptions.com
  • 14. START: Adverse Events  START: No difference in risk of selected adverse events[1,2] 1. INSIGHT START Group. N Engl J Med. 2015;373:795-807. 2. Lundgren J, et al. IAS 2015. Abstract MOSY0302.. Other Secondary Endpoints[1,2] Immediate ART (n = 2326) Deferred ART (n = 2359) HR (95% CI) P Value n n/100 PY n n/100 PY Grade 4 event 73 1.06 73 1.05 1.01 (0.73-1.39) .97 Unscheduled hospitalization 262 4.02 287 4.40 0.91 (0.77-1.08) .28 Grade 4 event, unscheduled hospitalization, or death from any cause 283 4.36 311 4.78 0.91 (0.77-1.07) .25 Slide credit: clinicaloptions.com
  • 15. TEMPRANO: Immediate vs Deferred ART Initiation and IPT Delivery for African Pts TEMPRANO ANRS 12136 Study Group. N Engl J Med. 2015;373:808-822. Mos From Randomization CumulativeProbability ofDeathorSevere HIV-RelatedIllness(%) 25 20 15 10 5 0 0 6 12 18 24 30 Deferred ART Deferred ART + IPT Immediate ART Immediate ART + IPT 30-Mo Probability, % 14.1 8.8 7.4 5.7 Slide credit: clinicaloptions.com
  • 16. DHHS Recommendations for Early HIV Infection  ART recommended for early HIV infection – Although no definitive data confirming whether this approach has long-term virologic, immunologic, or clinical benefits  ART recommended for pregnant women with early HIV infection – To prevent perinatal transmission  ART can start before drug resistance test results are available – Boosted PIs + 2 NRTIs recommended to prevent resistance in this setting DHHS Guidelines. November 2015 Slide credit: clinicaloptions.com
  • 17. HPTN 052: ART for Prevention of HIV Transmission in Serodiscordant Couples  International, randomized, controlled trial Stable, healthy, sexually active, HIV-discordant couples with CD4+ cell count 350-550 cells/mm3 (N = 1763 couples) Early ART Arm Initiate ART immediately (n = 886 couples) Delayed ART Arm Initiate ART at CD4+ cell count ≤ 250 cells/mm3 or at development of AIDS-defining illness (n = 877 couples) Cohen MS, et al. IAS 2015. Abstract MOAC0101LB. Slide credit: clinicaloptions.com
  • 18. HPTN 052: Reduced Risk of Partner Infection  ART offered to all index pts in delayed ART arm from May 2011 after interim results – 84% of pts in delayed ART arm had initiated ART at Yr 1 and 98% prior to study closure  No linked HIV transmissions observed when index participant stably suppressed on ART Partner Infections, n (rate/100 PY) Overall (April 2005 - May 2015) Early (4314 PY F/U) Delayed (4180 PY F/U) All 19 (0.44) 59 (1.41) Linked 3 (0.07) 43 (1.03) Risk Reduction With Early ART, % All infections 69 -- Linked infections 93 -- Cohen MS, et al. IAS 2015. Abstract MOAC0101LB. Slide credit: clinicaloptions.com
  • 19. Which regimen would you recommend? A. Dolutegravir/abacavir/ lamivudine B. Elvitegravir/cobicistat/ tenofovir DF/emtricitabine C. Elvitegravir/cobicistat/ tenofovir alafenamide/emtricitabine D. Efavirenz/ tenofovir DF/emtricitabine E. Rilpivirine/ tenofovir DF/emtricitabine F. Dolutegravir + tenofovir DF/emtricitabine G. Raltegravir + tenofovir DF/emtricitabine H. Darunavir + ritonavir + tenofovir DF/emtricitabine I. I would recommend a different regimen  22-yr-old black man with recent HIV seroconversion, multiple sex partners  HIV-1 RNA 2130 copies/mL, CD4+ cell count 726 cells/mm3 , no medical problems  Labs normal, HIV GT WT, HBV immune, HCV negative, HLA-B*5701 negative
  • 20. Discussion Question: Would your choice change if the pt . . .  Was HLA-B*5701 positive?  Was HBsAg positive?  Had high viral load?  Was a woman?  22-yr-old black man with recent HIV seroconversion  HIV-1 RNA 2130 copies/mL, CD4+ cell count 726 cells/mm3  No medical problems  Labs normal, HIV GT WT, HBV immune, HCV negative, HLA-B*5701 negative  Multiple sex partners
  • 21. DHHS, IAS-USA, EACS Guidelines: Recommended Regimens for First-line ART  Recommendations may differ based on baseline viral load, CD4+ count, CrCl, eGFR, HLA-B*5701 status, HBsAg status, and osteoporosis status  Publication of these guidelines preceded the availability of DTG/ABC/3TC as a single-tablet regimen 1. DHHS Guidelines. November 2015. 2. Günthard HF, et al. JAMA. 2014;312:410-425. 3. EACS Guidelines. October 2015. Class DHHS[1] IAS-USA[2] EACS[3] INSTI  DTG/ABC/3TC  DTG + TDF/FTC  EVG/COBI/TDF/FTC  EVG/COBI/TAF/FTC  RAL + TDF/FTC  DTG + ABC/3TC  DTG + TDF/FTC  EVG/COBI/TDF/FTC  RAL + TDF/FTC  DTG/ABC/3TC  DTG + TDF/FTC  EVG/COBI/TDF/FTC  RAL + TDF/FTC Boosted PI  DRV + RTV + TDF/FTC  DRV + RTV + TDF/FTC  ATV + RTV + TDF/FTC  ATV + RTV + ABC/3TC  DRV + RTV + TDF/FTC NNRTI  EFV/TDF/FTC  EFV + ABC/3TC  RPV/TDF/FTC  RPV/TDF/FTC Slide credit: clinicaloptions.com
  • 22. ACTG 5257: Open-Label ATV + RTV vs RAL vs DRV + RTV in First-line ART  Primary endpoints – Virologic failure: time to HIV-1 RNA > 1000 copies/mL (at Wk 16 or before Wk 24) or > 200 copies/mL (at or after Wk 24) – Tolerability failure: time to discontinuation of randomized component for toxicity  Composite endpoint: the earlier occurrence of either VF or TF in a given participant  Switch of regimens allowed for tolerability Lennox JL, et al. Ann Intern Med. 2014;161:461-471. ART-naive pts with HIV-1 RNA ≥ 1000 copies/mL (N = 1809) ATV + RTV 300 + 100 mg QD + TDF/FTC (n = 605) RAL 400 mg BID + TDF/FTC (n = 603) Stratified by HIV-1 RNA < or ≥ 100,000 copies/mL, participation in metabolic substudy, CV risk DRV + RTV 800/100 mg QD + TDF/FTC (n = 601) Wk 96 after last pt enrolled Slide credit: clinicaloptions.com
  • 23. ACTG 5257: Primary Endpoint Analyses at Wk 96  Regimens equivalent in time to VF Lennox JL, et al. Ann Intern Med. 2014;161:461-471.  Significantly greater incidence of treatment failure with ATV + RTV vs RAL or DRV + RTV – In part due to high frequency of hyperbilirubinemia*  Considering both efficacy and tolerability, RAL superior to either boosted PI  DRV + RTV superior to ATV + RTV Virologic Failure Tolerability Failure Composite Endpoint Difference in 96-Wk Cumulative Incidence (97.5% CI) 0-10 10 20 ATV + RTV vs RAL 3.4% (-0.7 to 7.4) DRV + RTV vs RAL 5.6% (1.3-9.9) ATV + RTV vs DRV + RTV -2.2% (-6.7 to 2.3) ATV + RTV vs DRV + RTV 9.2% (5.5-12.9) 0-10 10 20 ATV + RTV vs RAL 12.7% (9.4-16.1) DRV + RTV vs RAL 3.6% (1.4-5.8) Favors RAL Favors DRV + RTV 0-10 10 20 ATV + RTV vs RAL 14.9% (10.2-19.6) DRV + RTV vs RAL 7.5% (3.2-11.8) ATV + RTV vs DRV + RTV 7.5% (2.3-12.7) Favors RAL Favors DRV + RTV Favors RAL *Pts were allowed to switch regimens and remain on study. Slide credit: clinicaloptions.com
  • 24. Study 103: EVG/COBI/TDF/FTC Noninferior to ATV + RTV + TDF/FTC Through Wk 144 Outcomes at Wk 144[3] EVG/COBI/ TDF/FTC ATV + RTV + TDF/FTC Treatment- related d/c, % 6 9 Virologic failure, % 8 7 Mean CD4+ cell count increase, cells/mm3 280 293 Clumeck N, et al. J Acquir Immune Defic Syndr. 2014;65:e121-124. EVG/COBI/TDF/FTC (n = 353) ATV + RTV + TDF/FTC (n = 355) Δ: 2.7% (-2.1 to 7.5) Δ: 1.1% (-4.5 to 6.7) Wk 48 Wk 144 78 75 0 20 40 60 80 100 90 87 Δ: 3.1% (-3.2 to 9.4) 83 82 Wk 96 Slide credit: clinicaloptions.com
  • 25. WAVES: EVG/COBI/TDF/FTC vs ATV + RTV + TDF/FTC in Tx-Naive Women  International, randomized, double-blind phase III trial  Pts generally well matched at baseline – Pts with HIV-1 RNA > 100,000 copies/mL: EVG/COBI/TDF/FTC arm 24%; ATV + RTV + TDF/FTC arm 25% Squires K, et al. IAS 2015. Abstract MOLBPE08. EVG/COBI/TDF/FTC QD + Placebos for ATV, RTV, and TDF/FTC QD (n = 289) ATV + RTV + TDF/FTC QD + Placebo for EVG/COBI/TDF/FTC QD (n = 286) HIV-infected women with HIV-1 RNA ≥ 500 copies/mL; no previous ART; and eGFR ≥ 70 mL/min (N = 575) Wk 48 Open-label extension ATV 300 mg; RTV 100 mg; TDF/FTC 300/200 mg; EVG/COBI/TDF/FTC 150/150/300/200 mg Slide credit: clinicaloptions.com
  • 26. WAVES: EVG/COBI/TDF/FTC Superior to ATV + RTV + TDF/FTC At Wk 48  No significant differences between arms in change from BL for eGFR, spine or hip BMD, LDL or HDL cholesterol, total cholesterol to HDL ratio, or triglycerides  Significantly greater increase in total cholesterol with EVG/COBI/ TDF/FTC  Lower rate of discontinuations due to AEs with EVG/COBI/ TDF/FTC vs ATV + RTV + TDF/FTC (2.4% vs 7.0%) Squires K, et al. IAS 2015. Abstract MOLBPE08. Wk48HIV-1RNA<50c/mL(%) 100 80 60 40 20 0 Overall ≤ 100,000 > 100,000 HIV-1 RNA (copies/mL) EVG/COBI/TDF/FTC ATV + RTV + TDF/FTC 87 81 86 82 90 78 n = 289 286 220 214 69 72 Emergent Resistance EVG/COBI/FTC/TDF (n = 289) ATV+RTV + TDF/FTC (n = 286) Resistance analysis population 19 21 Developed resistance mutations to study drugs 0 3 Slide credit: clinicaloptions.com
  • 27. SINGLE: DTG + ABC/3TC Superior to EFV/TDF/FTC in Tx-Naive Pts Through Wk 144  Emergent resistance in those with VF: 0/39 (DTG) vs 7/33 (EFV) Virologic Success*[1] Virologic Nonresponse[2] No Virologic Data[2] Pts(%) Favors EFV/TDF/FTC 95% CI for Difference† 0% Wk 48 Wk 96 Wk 144 7.4% 8.0% 8.3% 2.5% 2.3 % 2% 14.6% 13.8% 12.3% Favors DTG+ABC/3TC 15% 1. Walmsley S, et al. J Acquir Immune Defic Syndr. 2015;70:515-519. 2. Pappa K, et al. ICAAC 2014. Abstract H-647a. 88 81 80 72 71 63 5 6 7 8 10 7 7 13 12 20 30 18 100 80 60 40 20 0 DTG + ABC/3TC QD (n = 414) EFV/TDF/FTC QD (n = 419) Wk 48 96 144 Wk 48 96 144 Wk 48 96 144 *HIV-1 RNA < 50 copies/mL as defined by FDA Snapshot algorithm. † -10% noninferiority margin. -5% Slide credit: clinicaloptions.com
  • 28. SPRING-2: DTG + 2 NRTIs Noninferior to RAL + 2 NRTIs Through Wk 96 Outcomes at Wk 96[2] DTG + NRTIs RAL + NRTIs D/c for AEs or death, % 2 2 Virologic nonresponse, % 5 10 Mean CD4+ cell count increase, cells/mm3 276 264 HIV-1RNA<50copies/mL(%) 88 85 DTG 50 mg QD (n = 411) RAL 400 mg BID (n = 411) 0 20 40 60 80 100 81 76 Wk 48[1] Wk 96[2] 1. Raffi F, et al. Lancet. 2013;381:735-743. 2. Raffi F, et al. Lancet Infect Dis. 2013;13:927-935. 361/ 411 351/ 411 332/ 411 314/ 411 Δ 4.5% (-1.1% to 10.0%) Δ 2.5% (-2.2% to 7.1%) n/N = Slide credit: clinicaloptions.com
  • 29. 40 FLAMINGO: DTG Superior to DRV + RTV in ART-Naive Pts Through Wk 96 Virologic Success Virologic Nonresponse No Virologic Data Favors DRV + RTV 95% CI for Difference 0%-12% Wk 48 Wk 96 Subjects(%) Favors DTG 25% DTG + 2 NRTIs (n = 242) DRV + RTV + 2 NRTIs (n = 242) Molina J-M, et al. Lancet HIV 2015;2:e127–36. 7.1% 12.4% 0.9% 4.7% 20.2% 13.2% Wk 48 Wk 96 Wk 48 Wk 96 Wk 48 Wk 96 100 80 60 20 0 83 90 80 68 6 7 8 12 4 10 12 21 15 18 19 28 10 24 29 50217 200 194 164n = Slide credit: clinicaloptions.com
  • 30. Studies 104/111: Tenofovir Alafenamide Fumarate vs TDF in Treatment-Naive Pts  Parallel, randomized, double-blind, active-controlled phase III studies  Primary endpoint: HIV-1 RNA < 50 c/mL at Wk 48, as defined by FDA Snapshot algorithm EVG/COBI/FTC/TAF* single-tablet regimen (n = 866) EVG/COBI/FTC/TDF† single-tablet regimen (n = 867) Treatment-naive HIV-infected pts with HIV-1 RNA ≥ 1000 copies/mL, eGFR ≥ 50 mL/min (N = 1733) Stratified by HIV-1 RNA, CD4+ cell count, geographic region Wk 48 Primary endpoint Wk 144 *150/150/200/10 mg once daily. † 150/150/200/300 mg once daily. Sax PE, et al. Lancet. 2015;385:2606-2615. Slide credit: clinicaloptions.com
  • 31. Studies 104/111: TAF Noninferior to TDF at Wk 48  EVG/COBI/FTC/TAF was noninferior to EVG/COBI/FTC/TDF at Wk 48 in each study: 93% vs 92% (Study 104); 92% vs 89% (Study 111) – Race not significant predictor of virologic efficacy in multivariate analysis  Declines in eGFR and in hip and spine BMD significantly less in TAF arm  Discontinued for AE, death, or missing data. 1. Sax PE, et al. Lancet. 2015;385:2606-2615. 2. Wohl D, et al. ID Week 2015. Abstract 1073. No Data Virologic Success Virologic Failure Pts(%) 92 90 EVG/COBI/FTC/TAF (n = 866) EVG/COBI/FTC/TDF (n = 867) 0 20 40 60 80 100 4 4 4 6 n = 800 784 Favors TAF 0 4.7%-0.7% 2.0% Treatment Difference (95% CI) -12% +12% Favors TDF Virologic Outcome Slide credit: clinicaloptions.com
  • 32. Studies 104/111: Renal Outcomes With TAF vs TDF in Black vs Nonblack Pts  In black pts, decrease in median eGFR significantly smaller with TAF vs TDF  Less spine and hip BMD loss with TAF vs TDF both in black and nonblack pts Wohl D, et al. ID Week 2015. Abstract 1073. Black Pts EVG/COBI/FTC/TAF Nonblack Pts EVG/COBI/FTC/TDF 15 10 5 0 -5 -10 MeaneGFRChange(mL/min) -15 -20 -25 -30 15 10 5 0 -5 -10 -15 -20 -25 -30 0 2 4 8 12 16 24 36 48 Wk 0 2 4 8 12 16 24 36 48 Wk Slide credit: clinicaloptions.com
  • 33. Potential Advantages and Disadvantages of Single-Tablet Regimens Advantages Disadvantages  Simplicity  Convenience  Fewer copays  Reduces selective nonadherence to components of regimen  Inability to adjust dosages of components if needed due to drug– drug interactions or tolerability issues, eg, renal insufficiency  Not available for all ART regimens  Not available for all NRTI pairings Slide credit: clinicaloptions.com
  • 34. Available Single-Tablet Regimens Agent Type Year of FDA Approval Efavirenz/tenofovir DF/ emtricitabine (EFV/TDF/FTC) NNRTI + dual NRTI 2006 Rilpivirine/tenofovir DF/ emtricitabine (RPV/TDF/FTC) NNRTI + dual NRTI 2011 Elvitegravir/cobicistat/ tenofovir DF/emtricitabine (EVG/COBI/TDF/FTC)* INSTI + booster + dual NRTI 2012 Dolutegravir/abacavir/lamivudine (DTG/ABC/3TC)* INSTI + dual NRTI 2014 Elvitegravir/cobicistat/ tenofovir alafenamide/emtricitabine (EVG/COBI/TAF/FTC)* INSTI + booster + dual NRTI 2015 *DHHS recommended regimen for initial ART. Slide credit: clinicaloptions.com
  • 35. Take-Home Points  Randomized trial data support ART initiation in pts with CD4+ cell count > 500 cells/mm3  ART guidelines recommend ART for all pts regardless of CD4+ cell count  Recommended regimens for ART initiation have been revised – NNRTIs removed from DHHS first-line options – EVG/COBI/TAF/FTC added to DHHS first-line options, safe in pts with CrCl ≥ 30 mL/min Slide credit: clinicaloptions.com
  • 36. Case 2: HIV-Infected Patient With Common Comorbidities
  • 37. Case 2: 53-Yr-Old Man With HIV Infection and Multiple Medical Problems  53-yr-old man presents with newly diagnosed HIV infection  Tested for HIV infection by his PCP, who has been treating him for hyperlipidemia and DM for 12 yrs  Hospitalized 2 yrs earlier for chest pain and diagnosed with NSTEMI  Reports he has been better in the last yr at sticking to his medical regimen and now rarely misses a dose of his prescribed medications (metformin, glipizide, aspirin, metoprolol, and atorvastatin)  He is an exsmoker and denies use of illicit drugs Slide credit: clinicaloptions.com
  • 38. Case 2: Laboratory Analysis  CD4+ count 423 cells/mm3 (27%), HIV-1 RNA 69,554 copies/mL  HIV genotype K103N (resistant to EFV, NVP, DLV)  HLA-B*5701 negative  Cr/BUN 1.6/20, eGFRCG 53 mL/min  ALT/AST normal, HBV immune, HCV negative  Last recorded A1c 6.5; last LDL 105 mg/dL  His hyperlipidemia and DM are relatively well controlled  He is interested in starting ART Slide credit: clinicaloptions.com
  • 39. Which NRTI combination would you recommend? A. Abacavir/lamivudine B. Tenofovir DF/emtricitabine C. Tenofovir alafenamide/emtricitabine (as E/C/F/TAF) D. I would use a different NRTI combination E. I would use lamivudine or emtricitabine without other NRTIs F. I would not use NRTIs in this pt G. Unsure  53-yr-old man recently diagnosed with HIV infection  HIV-1 RNA 69,554 copies/mL, CD4+ count 423 cells/mm3  Hyperlipidemia and DM controlled on medication, history of NSTEMI  Cr/BUN 1.6/20, eGFR 53 mL/min, A1c 6.5, LDL 105 mg/dL  HIV GT: K103N, HBV immune, HCV negative, HLA-B*5701 negative
  • 40. Which other agents would you use in the regimen? A. Boosted PI B. INSTI C. Boosted PI + INSTI D. Other E. Unsure  53-yr-old man recently diagnosed with HIV infection  HIV-1 RNA 69,554 copies/mL, CD4+ count 423 cells/mm3  Hyperlipidemia and DM controlled on medication, history of NSTEMI  Cr/BUN 1.6/20, eGFR 53 mL/min, A1c 6.5, LDL 105 mg/dL  HIV GT: K103N, HBV immune, HCV negative, HLA-B*5701 negative
  • 41. Rising Rates of Comorbidities at HIV Diagnosis in USA Pts(%) Meyer N, et al. IAS 2015. Abstract MOPEB157. Medicare (> 65 Yrs) 100 80 60 40 20 0 All CV HTN DM Renal 2003 (n = 177; mean age: 72.2 yrs) 2013 (n = 436; mean age: 72.9 yrs) Pts(%) Medicaid 100 80 60 40 20 0 All CV HTN DM Renal 2003 (n = 3008; mean age: 34.7 yrs) 2013 (n = 1632; mean age: 39.2 yrs) Slide credit: clinicaloptions.com
  • 42. IAS-USA: Recommendations for Initial ART in the Settings of Specific Conditions  In pts with or at high risk of CVD, consider avoiding ABC, LPV/RTV, or FPV + RTV  In pts with reduced renal function, TDF should generally be avoided, especially with a boosted PI  In pts at elevated fracture risk (eg, HCV coinfection, postmenopausal women, osteoporosis), it may be prudent to avoid TDF, especially with a boosted PI Günthard HF, et al. JAMA. 2014;312:410-425. Slide credit: clinicaloptions.com
  • 43. 1.80 1.60 1.40 1.20 1.00 0.00 D:A:D: Cumulative Exposure to ARVs Associated With Increased CKD Risk CKD Risk by Yrs of ARV Exposure, IRR (95% CI) Drug 1 Yr 2 Yrs 5 Yrs TDF 1.12 (1.06-1.18) 1.25 (1.12-1.39) 1.74 (1.33-2.27) ATV+ RTV 1.27 (1.18-1.36) 1.61 (1.40-1.84) 3.27 (2.32-4.61) LPV/ RTV 1.16 (1.10-1.22) 1.35 (1.21-1.50) 2.11 (1.62-2.75) Mocroft A, et al. CROI 2015. Abstract 142. Relationship Between Increasing Exposure to ARVs and CKD ATV + RTV LPV/RTVTDF Multivariate (on treatment) Multivariate (TDF censored) Univariate Multivariate Slide credit: clinicaloptions.com
  • 44. Summary of Key Analyses Showing ABC Associated With Risk of MI Study Study Design Age, Yrs (Range) Event (n) Pts, N TDF CV Effect ABC CV Effect Time on ABC, Mos Risk of MI (95% CI) D:A:D[1] Cohort 40 (35-47) MI, validated (387) 22,625 No Yes ≥ 6 2.04 (1.66-2.51) D:A:D 2015[2] Cohort 39 (33-46) MI (493) 32,663 Yes Current 1.47 (1.26-1.71) SMART[3] RCT 45 (39-51) MI, validated (19) 2752 No Yes Current 4.3 (1.4-13.0) STEAL[4] RCT 45.7 ±8.8 MI (4) 357 No Yes 96 2.79* (1.76-4.43) QPHID[5] CC 47 (22-67) MI (125) 7053 No Yes Any 1.79 (1.16-2.76) Danish[6] Cohort 39 (33-47) MI (67) 2952 No Yes > 6 2.00 (1.07-3.76) VA (Choi)[7] Cohort 46 CVD event (501) 10,931 No Yes Recent 1.64 (0.88-3.08) Swiss[8] Cohort Not given CVD event (365) 11,856 No Yes Recent 4.06† (2.24-7.34) MAGNIFICENT[9] CC 50 (22-85.5) CVD event (571) 1875 No Yes Current 1.56 (1.17-2.07) NA-ACCORD[10] Cohort MI, validated (301) 16,733 Yes ≤ 6 1.33 References in slidenotes Slide credit: clinicaloptions.com *Risk for serious non-AIDS events (most common was CVD, including MI); HR for CVD with TDF vs ABC: 0.12 (95% CI: 0.02-0.98; P = .048). † Risk for CVD event, including MI, invasive CV procedure, or CV-related death.
  • 45. Summary of Key Analyses Showing ABC NOT Associated With Risk of MI References in slidenotes Study Study Design Age, Yrs (Range) Event (n) Pts, N TDF CV Effect ABC CV Effect Time on ABC, Mos Adj Risk of MI (95% CI) FHDH[1] CC 47 (41-54) MI (289) 74,958 No No < 12/ recent 1.27* (0.64-2.49) ALLRT/ ACTG[2] Cohort 37 (26-51) MI (36) 5056 No No 72 0.6 (0.3 -1.4) VA[3] Cohort 46 MI (278) 19,424 No No Per 12 1.18 (0.92-1.50) FDA[4] Meta- analysis of RCTs 36-42 MI (46) 9868 No No 19 1.02 (0.56-1.84) NA-ACCORD[5] Cohort MI, validated (301) 16,733 No ≤ 6 1.33 *Without adjustment for cocaine use OR: 2.01 (1.11-3.64). Slide credit: clinicaloptions.com
  • 46. Subanalysis of 104/111: TAF Noninferior to TDF at Wk 48 in Pts Aged 50 Yrs or Older  Mean eGFR decline and mean % BMD decrease from baseline to Wk 48 significantly lower with TAF vs TDF (eGFR: P = .01; spine: P = .01; hip: P < .001) Daar ES, et al. ID Week 2015. Abstract 1074. 100 80 60 40 20 0 Virologic Success Virologic Failure No Data EVG/COBI/FTC/TAF (n = 89) EVG/COBI/FTC/TDF (n = 114) 94 91 3 4 2 4 -14 -10 -6 -2 2 6 10 14 -5.2 3.5 12.2 Favors TAFFavors TDF Treatment Difference (95% CI)Virologic Outcome Pts(%) Slide credit: clinicaloptions.com
  • 47. Phase III Trial of Stable Switch to EVG/COBI/TAF/FTC in Pts With CKD  Multicenter, single-arm, phase III switch study  Primary safety endpoint: eGFR at Wk 24  Baseline characteristics – Median age 58 yrs, median eGFR 56 mL/min, clinically significant albuminuria 49%, median CD4+ count 632 cells/mm3 , pre-switch TDF 65%, HTN 40%, DM 14% EVG/COBI/TAF/FTC (150/150/10/200 mg QD) single-tablet regimen HIV-infected pts with HIV-1 RNA < 50 copies/mL for ≥ 6 mos, CD4+ cell count ≥ 50 cells/mm3 , and eGFR 30-69 mL/min (N = 242) Wk 24 Primary endpoint Wk 96 Gupta S, et al. IAS 2015. Abstract TUAB0103. Slide credit: clinicaloptions.com
  • 48. Change in Renal Function Following Switch to EVG/COBI/TAF/FTC  No change in actual GFR at Wk 24  In pts on TDF, tubular proteinuria improved after switch MedianChangeFrom Baseline eGFRCG mL/min eGFRCKD-EPI Cr mL/min/1.73m2 eGFRCKD-EPI cys C mL/min/1.73m2 TDF at BL Non-TDF at BLAll pts -0.6 +0.2 -1.8 -1.8* -1.5 -2.7* +1.6* -1.4 +2.7**P < .05 Gupta S, et al. IAS 2015. Abstract TUAB0103. Change in eGFR From Baseline to Wk 48 10 0 -10 Slide credit: clinicaloptions.com
  • 49. Considerations for Pts With Renal Impairment CrCl (mL/min) EVG/COBI/FTC/TAF[1] EVG/COBI/FTC/TDF[2] ≥ 70 No adjustment needed No adjustment needed 50-70 No adjustment needed Initiation not recommended 30-49 No adjustment needed Initiation not recommended; Discontinue if CrCl declines to this level during treatment < 30 Initiation not recommended Initiation not recommended Discontinue if CrCl declines to this level during treatment 1. EVG/COBI/FTC/TAF [package insert]. 2. EVG/COBI/FTC/TDF [package insert]. Slide credit: clinicaloptions.com
  • 50. If you were considering an ABC, TDF, and TAF– sparing regimen, which would you most strongly consider? A. Boosted PI monotherapy B. Boosted PI + INSTI ± 3TC (or FTC) C. Boosted PI + 3TC (or FTC) D. DTG + 3TC E. I would not consider an ABC-, TDF-, and TAF-sparing regimen in this type of pt F. Other  53-yr-old man recently diagnosed with HIV infection  HIV-1 RNA 69,554 copies/mL, CD4+ count 423 cells/mm3  Hyperlipidemia and DM controlled on medication, history of NSTEMI  Cr/BUN 1.6/20, eGFR 53 mL/min, A1c 6.5, LDL 105 mg/dL  HIV GT: K103N, HBV immune, HCV negative, HLA-B*5701 negative
  • 51. < 100,000 c/mL ≥ 100,000 c/mL n = 530 n = 275 BL HIV-1 RNA NEAT: RAL + DRV/RTV Noninferior to TDF/FTC + DRV/RTV in Naive Pts at 96 Wks  Randomized, open-label phase III study of DRV/RTV + RAL vs DRV/RTV + TDF/FTC in ART-naive pts Raffi F, et al. Lancet. 2014;384:1942-1951. Overall N = 805 Primary Endpoint at Wk 96: Adjusted Difference in Proportion of Pts With Failure (RAL - TDF/FTC [95% CI]) -10 0 10 20 30 RAL TDF/FTC Adjusted Difference Estimate (95% CI) 17.8 13.8 4.0 (-0.8 to 8.8) 7.4 36.8 7.3 27.3 0.1 (-3.8 to 4.0) 9.6 (-0.1 to 20.1) 43.2 13.7 20.9 12.3 22.3 (7.4 to 37.1) 1.4 (-3.5 to 6.3) < 200/mm3 ≥ 200/mm3 n = 123 n = 682 BL CD4+ cell count Slide credit: clinicaloptions.com
  • 52. GARDEL: Dual ART Noninferior to Triple ART in Tx-Naive Pts at Wks 48 and 96  Phase III, international, open-label, randomized study  Safety and tolerability also similar between treatment arms Virologic Success Virologic Nonresponse D/C due to AE or Death D/C for Other Reasons 1. Cahn P, et al. EACS 2015. Abstract 961. 2. Cahn P, et al. Lancet Infect Dis. 2014;14:572-80. Wk 48 difference: +4.6% (95% CI: -2.2 to 11.8; P = .171) Wk 96 difference: +5.9% (95% CI: -2.3 to 14.1; P = .165) 100 80 60 40 20 0 Pts(%) LPV/RTV + 3TC LPV/RTV + 3TC or FTC + NRTI 4.7 5.9 3 5 6 82.4 2.1 0.6 2.8 6.6 10.6 88.3 83.7 90.3 84.4 Wk: 96[1] 48[1] 96[1] 48[2] 96[1] 48[2] 96[1] 48[2] Slide credit: clinicaloptions.com
  • 53. PADDLE: Dolutegravir + Lamivudine in Treatment-Naive Pts  Open-label, single-arm phase IV exploratory trial  BL RNA: median 24,128 copies/mL; IQR 11,686-36,794 copies/mL  20 of 20 pts met primary endpoint of HIV-1 RNA < 50 copies/mL at Wk 24 (ITT-e, FDA snapshot analysis) – Including 4 pts with BL HIV-1 RNA > 100,000 copies/mL – All pts virologically suppressed by Wk 8 Treatment-naive pts with HIV-1 RNA 5000-100,000 copies/mL; CD4+ ≥ 200 cells/mm3 ; HBsAg negative (N = 20) DTG 50 mg QD + 3TC 300 mg QD (N = 20*) *Pts enrolled in 2 cohorts of 10 pts. Second cohort enrolled following confirmation of first cohort success at Wk 8. Figueroa MI, et al. EACS 2015. Abstract 1066. Slide credit: clinicaloptions.com
  • 54. Recommendations on the Use of NRTI- Sparing Regimens in First-line ART  Regimens using < 2 NRTIs should only be used in pts who cannot take ABC or TDF  These regimens can be considered when ABC or TDF cannot be used: – DRV + RTV + RAL (only for pts with HIV-1 RNA < 100,000 copies/mL and CD4+ cell count > 200 cells/mm3 ) – LPV/RTV (BID) + 3TC (BID) DHHS Guidelines. November 2015 Slide credit: clinicaloptions.com
  • 56. Case 3: 40-Yr-Old Woman With An OI  A 40-yr-old woman presents to the ED with L arm weakness, difficulty walking, and dysarthric speech  Prior history of positive HIV serology, but she is homeless and has neither entered into care for HIV nor received ART  CT scan shows R frontoparietal mass with extensive edema; no hydrocephalus or midline shift  Toxoplasma IgG is positive  Pt is started on sulfadiazine, pyrimethamine and folinic acid with improvement in her neurological status Slide credit: clinicaloptions.com
  • 57. Case 3, Laboratory Analysis  HIV-1 RNA 175,000 copies/mL, CD4+ count 65 cells/mm3 (4%)  HIV genotype is pending  Coinfection with HCV (HCV RNA positive) and HBV (HBsAg positive)  HLA-B*5701 negative  No known history of heart disease or renal insufficiency  History of injection drug use  She smokes 1 pack/day and uses methamphetamine and opiates when she can get them Slide credit: clinicaloptions.com
  • 58. When would you recommend starting antiretroviral therapy? A. Start ART within 2-3 weeks B. Defer ART until the pt completes treatment for toxoplasmic encephalitis C. Defer ART until the pt is in a stable living situation D. Defer ART until the pt starts a drug treatment program E. Unsure  40-yr-old woman with untreated HIV infection and new dx of toxoplasmic encephalitis  HIV-1 RNA 175,000 copies/mL, CD4+ cell count 65 cells/mm3  HIV GT pending, HBsAg positive, HCV RNA positive, HLA-B*5701 negative  History of injection drug use  Homeless
  • 59. Favors Deferred ART Zolopa AR, et al. PLoS ONE. 2009;4:e5575. ACTG 5164: Immediate vs Deferred ART in Pts With Acute Opportunistic Infections Risk of AIDS Progression/Death by Entry Diagnoses, Log OR (95% CI) Total PCP Bacterial infection Other OI* Fungal Crypto Mycobacterial > 1 OI CD4+ < 50 CD4+ ≥ 50 Events, n/N 54/282 28/181 11/41 42/194 12/52 8/41 8/18 30/148 39/196 15/86 0 0.25 0.5 1.0 8.0 202.5 Favors Early ART *Includes 13 pts with toxoplasmosis Slide credit: clinicaloptions.com
  • 60. Recommendations for ART in Pts With Selected Opportunistic Infections Opportunistic Infection DHHS Recommendation for ART Pneumocystis pneumonia  Start ART within 2 wks of PCP diagnosis Toxoplasma gondii encephalitis  Many clinicians start ART within 2-3 wks  Based on A5164 study, in which the 282 pts with OIs included 13 pts (5%) with toxoplasmosis Mycobacterium tuberculosis  Start ART within 2 wks if CD4+ < 50 cells/mm3 , by 8-12 wks for all others  Consider DDIs, adherence support Cryptosporidiosis  Start ART as part of OI management Cryptococcal meningitis  Consider delaying ART until after antifungal induction (2 wks) or induction/consolidation (10 wks) DHHS Guidelines. November 2015 Slide credit: clinicaloptions.com
  • 61. Which regimen would you recommend if starting ART prior to discharge? A. Dolutegravir/abacavir/lamivudine B. Dolutegravir + tenofovir DF/emtricitabine C. Elvitegravir/cobicistat/tenofovir DF/emtricitabine D. Elvitegravir/cobicistat/tenofovir alafenamide/emtricitabine E. Raltegravir + tenofovir DF/emtricitabine F. Boosted darunavir + tenofovir DF/emtricitabine G. NNRTI-based regimen H. Something else  40-yr-old woman with untreated HIV infection and new dx of toxoplasmic encephalitis  HIV-1 RNA 175,000 copies/mL, CD4+ cell count 65 cells/mm3  HIV GT pending, HBsAg positive, HCV RNA positive, HLA-B*5701 negative  History of injection drug use  Homeless
  • 62. Prevalence of Drug Resistance Mutations in Treatment-Naive Pts, 2000-2013  Baseline plasma samples from 4 phase III trials (GS 903, 934, 104, 111; N = 2531) – 1617 samples analyzed for integrase mutations – 2531 analyzed for protease or RT mutations  Substantial ↑ in prevalence of NNRTI resistance, modest ↑ in PI resistance  Stable prevalence of NRTI resistance (mostly TAMs) – M184V/I ≤ 0.2%; K65R ≤ 0.1%  Little evidence of transmitted INSTI resistance over period – Mostly T97A polymorphism 2000 (GS-903) 2003 (GS-934) 2013 (GS-104/GS- 111) 0 2 NNRTI 10 4 6 8 NRTI PI INSTI 0.5 1.0 0 4.2 8.7 3.2 2.6 2.6 1.2 2.4 2.9 1.4 Margot NA, et al. CROI 2014. Abstract 578. Slide credit: clinicaloptions.com
  • 63. Summary  Randomized trial data support ART initiation in pts with CD4+ cell count > 500 cells/mm3  ART guidelines recommend ART for all pts regardless of CD4+ cell count  Recommended regimens for ART initiation have been revised  ART selection should be individualized according to pt requirements, the evidence base, and practice guidance Slide credit: clinicaloptions.com
  • 64. Go Online for More CCO Coverage of HIV! Additional slidesets on contemporary management of HIV with expert faculty commentary Postconference Clinical Updates available following CROI, the International AIDS Conference, and IDWeek clinicaloptions.com/hiv

Hinweis der Redaktion

  1. Disclaimer: The materials published on the Clinical Care Options Web site reflect the views of the authors of the CCO material, not those of Clinical Care Options, LLC, the CME providers, or the companies providing educational grants. The materials may discuss uses and dosages for therapeutic products that have not been approved by the United States Food and Drug Administration. A qualified healthcare professional should be consulted before using any therapeutic product discussed. Readers should verify all information and data before treating patients or using any therapies described in these materials.
  2. Slidenotes may include transcripts from live presentations given by faculty other than those listed on this slide.
  3. Slidenotes may include transcripts from live presentations given by faculty other than those listed on this slide.
  4. HBV, hepatitis B virus; HCV, hepatitis C virus; WT, wild type. We have a 22-year-old man recently diagnosed with HIV infection. He&amp;apos;s a black man diagnosed after a routine screening. He was HIV uninfected at testing 6 months ago, no significant medical problems, not on any medications. His lipids, renal function, and liver function tests are normal. His CD4 count is a robust 726, viral load low—2130 copies—wild-type HIV genotype, hepatitis B immune, hepatitis C negative, HLA-B*5701 negative, and he says, “I am not sure if I want to start therapy now.”
  5. He expresses concerns about needing to take medications every day; he&amp;apos;s worried about adverse events. He doesn&amp;apos;t smoke; he has no family history of heart disease. He has multiple sexual partners, but he says he &amp;quot;almost always&amp;quot; uses condoms when he has sex. That means most of the time he doesn&amp;apos;t use condoms; that&amp;apos;s what that means.
  6. HBV, hepatitis B virus; HCV, hepatitis C virus; WT, wild type.
  7. ART, antiretroviral therapy; CVD, cardiovascular disease; DSMB, data and safety monitoring board; ESRD, end-stage renal disease. This is really one of the pivotal studies that we have seen in this last year—the START study—and this study has changed all of the HIV treatment guidelines. It was a study done by the INSIGHT Group—an international collaboration—and the design is people with CD4 counts &amp;gt; 500 and naive to therapy are going to get randomized to either start therapy right away or to delay the initiation of therapy until the CD4 count falls down &amp;lt; 350, or they get a clinical illness that would warrant the initiation of HIV treatment.   The primary outcomes for this study will be death and serious AIDS-defining illnesses—so not Candida esophagitis, rather things that you would really care about—and serious non-AIDS events. These would be non-AIDS–defining malignancies, cardiovascular events, kidney disease, and liver disease.
  8. ART, antiretroviral therapy; PY, patient-years. Here&amp;apos;s the primary outcome; this is the composite endpoint. You can see the number of events that have occurred, the proportion of individuals who&amp;apos;ve experienced an event. The mean follow-up here is about 3 years. And the immediate therapy group was 57% less likely to experience an event compared to the individuals whose treatment was delayed, so clearly an advantage.
  9. ART, antiretroviral therapy; PY, patient-years. Let&amp;apos;s look at this in a little bit more detail. The serious AIDS events occurred with relatively low frequency; the rates are &amp;lt; 1 event per 100 person-years. However, the protection from immediate therapy is 72%. If you look at what these events are, what do you think these events are? This is a study that&amp;apos;s going on in Africa; it&amp;apos;s a study that&amp;apos;s going on in Europe, Australia, in the United States. What do you think the most common opportunistic infection is in this study? Tuberculosis. And the rate of tuberculosis is cut in half if you start therapy early.
  10. ART, antiretroviral therapy; PY, patient-years. Now, what do you think the most common non-AIDS–defining events are in this study? Cancer. And we’re going to cut down the rate of cancer, of non-AIDS–defining malignancies, by 39% in people starting on therapy early. So not as much protection as with AIDS-defining events, but again a statistically significant benefit.
  11. ART, antiretroviral therapy; HL, Hodgkin’s lymphoma; NHL, non-Hodgkin’s lymphoma. Here are a few of the cancers that are occurring. We’re going to reduce cancers by immediate therapy—KS, lymphoma—not so much some of these other malignancies. While the benefit from AIDS-defining events was mostly driven by events happening in the developing world, the observation of less Kaposi’s sarcoma, less non-Hodgkin’s lymphoma, are non-AIDS–defining events that are happening in the developed world.  
  12. ART, antiretroviral therapy; PY, patient-years. Interestingly, there was no benefit from a cardiovascular perspective; so we didn’t cut down the rate of cardiovascular events—14 events in the people whose treatment was deferred, 12 in the people who start right away. That’s not a statistically significant difference. We’re also not going to have a statistically significant impact on grade 4 events, unscheduled hospitalizations, or the composite endpoint.   START is an important study showing the benefit of immediate therapy compared to delayed therapy. It’s a benefit that people see in the developing world; it’s a benefit that people experience in the developed world.  
  13. ART, antiretroviral therapy; IPT, isoniazid preventive therapy. The TEMPRANO study, a study done on the Ivory Coast, is another key study that was reported in 2015. To get into this study, people had to not have a condition that would require the initiation of therapy based upon the World Health Organization guidelines, and they couldn’t have a CD4 count &amp;gt; 800. There were 4 arms to this study. Naive-to-therapy patients were randomly assigned to defer the initiation of antiretroviral therapy. If they were randomized to defer therapy, that deferral would end when they had a condition that the World Health Organization would say would require treatment . The second group also got deferred therapy, but they got isoniazid preventive therapy (IPT). Group 3 got immediate treatment but no IPT, and group 4 got both antiretroviral therapy and 6 months of isoniazid.   Here are the outcomes for the whole cohort. The outcomes here are death, serious AIDS-defining conditions, non-AIDS–defining cancer, or invasive bacterial disease, an outcome that we’re a little bit less familiar with, but something that, in sub-Saharan Africa, is an issue. You can see there was an advantage of early therapy and also isoniazid preventive therapy. There’s also an added benefit for both interventions.  
  14. ART, antiretroviral therapy; DHHS, Department of Health and Human Services. As a result of these 2 studies, the World Health Organization changed their guidelines in September 2015 to endorse antiretroviral therapy for everybody aged 19 and above. These 2 studies have had a huge impact on how we think about when to initiate therapy. The most recently updated guidelines are those from the Department of Health and Human Services (DHHS). The other guidelines that we have in the US, the IAS-USA guidelines, are updated every other year, so we haven’t seen an update based upon these new studies; we’ll see that in July 2016 at the timing of the next International AIDS Society meeting.   The recommendation is to give therapy to everybody, including pregnant women of course. If somebody requires the initiation of therapy before a resistance test result has returned, you can start the person on a couple of NRTIs and a boosted protease inhibitor because, as you know, the prevalence of PI mutations in naive patients is relatively low, and the probability of developing resistance is lower on a boosted protease inhibitor if you experience virologic failure.  
  15. ART, antiretroviral therapy. The other study that influenced the notion that everybody needs to be treated is HPTN 052. This is a study where the benefit of therapy is not necessarily for the infected individual, but for public health. This study looked at immediate therapy vs delayed therapy, with the goal of preventing the transmission of virus to the partner of the individual who is participating in the study. If you’re not familiar, this was a couples study where the index case was randomized to start therapy or delay the initiation of therapy and the main outcome was infection rates in his or her partner.   This study was mostly done in the developing world; there were 2 sites in the United States—one in Boston and one in San Francisco—but they enrolled relatively few people because in the United States, most providers were already encouraging the early initiation of therapy in everybody, and therefore, you couldn’t find a lot of people who were willing to delay initiation. The enrollment CD4 count is a little bit lower in this study compared to others (ie, 350-550). Then for the delayed ART arm, you had to wait to initiate therapy until the CD4 count falls down &amp;lt; 250, which was something most providers were not comfortable with recommending when this study was done.   
  16. ART, antiretroviral therapy; F/U, follow-up; PY, patient-years. The main outcome of HPTN 052 was a reduction in transmission. When the data was initially published in 2011, there was only one documented linked transmission; a linked transmission means that the individual who was infected had their virus sequenced, and the virus that wound up in the partner was the same isolate. There were some unlinked transmissions, meaning somebody got infected outside of the couple’s relationship, but you don’t want to count that against the initial treatment or the delay in therapy. After the study was analyzed and we knew the outcomes, everybody who was in the deferred treatment group was given the opportunity to start therapy right away. With subsequent analysis, there were a total of 3 linked transmissions in the people who were started early.   All of these transmissions occurred before the infected individual had their viral load reduced to undetectable levels. No transmission occurred in this study when somebody’s viral load was undetectable. A couple of transmissions that aren’t described on this slide occurred in individuals who experienced virologic failure, and the transmission occurred after virologic rebound. Regardless, the initial goal of early vs delayed therapy resulted in &amp;gt; 90% rate of viral protection.   Furthermore, this was predominantly a heterosexual population (98%) that had access to condoms and education. So if people wanted to use condoms, they were readily available for use, but who knows how often they were used. I don’t think that we really know how much unprotected sex was going on.   
  17. HBV, hepatitis B virus; HCV, hepatitis C virus; WT, wild type. So remember our patient, that 22-year-old black man with recent seroconversion, multiple sex partners, viral load of 2130, CD4 count of 726, no medical problems, wild-type HIV genotype, HBV immune, HCV negative, and HLA-B*5701 negative?   Let me tell you why I might not want to treat this guy: because this guy said, “I’m not sure if I’m ready to start therapy.” Yes, the data says he’d be better off on therapy. Yes, he has a lot of sexual partners, and certainly they would be better off if he was on therapy with an undetectable viral load. However, if this guy tells me “I’m not quite sure if I am ready,” and he’s 22 years old, in my experience with adolescents, these are the patients who are most likely to mess up and then disappear on me, so these are the patients where I’m going to be a little gun shy. I like to work with people to slowly convince them that they need therapy, that they’d be better off on therapy, that it’s a good thing, that they can do it. I like to help them believe that they can do it. I like them to imagine how they can be successful, especially because this is a young guy who has never taken any medications. Although I believe that he would benefit from therapy, I would wait.   Even if the patient’s viral load was elevated, I still may wait, but I don’t think I’d feel as comfortable waiting as long. I would probably say, “Okay, you don’t need to walk out of the office here today with a prescription, or the next time I see you in a month, but your viral load is high and that means that this nice CD4 count you have going is not going to stay nice for a very long period of time. It also means that the probability of you transmitting your infection to somebody you’re having sex with is considerably higher.” This patient’s viral load is sort of near the threshold of a study that was done in Rakai a number of years ago that Tom Quinn was the first author on. In that study, there were no transmissions—this was heterosexual transmissions—if somebody’s viral load was &amp;lt; 1500. Our patient’s viral load is close to that. So the risk of him transmitting his infection is relatively lower because his viral load is lower. However, once his viral load is &amp;gt; 100,000, now I’m nervous.   Test-and-treat programs are being piloted where people are diagnosed and then given prescriptions right after learning that they have HIV, which I think is too fast. I think that a lot of people who learn that they have HIV infection need a moment to think before they’re rushed into treatment.   That all being said, let’s consider the question: What regimen would you recommend for our first case study patient?   Maybe (option C), fixed-dose combination elvitegravir/cobicistat/emtricitabine with a new formulation of tenofovir alafenamide? I’ll eventually show you that this combination offers a lower risk for renal events or bone mineral toxicity, that is if you can get it to your patient.   What about (option E), tenofovir/emtricitabine/rilpivirine? Is there a reason, anything in his history, to not choose this for our patient? First, it’s not a recommended first-line regimen by the DHHS guidelines. You also don’t want to give this combination to somebody with a baseline viral load of &amp;gt; 100,000 or if somebody’s on a proton pump inhibitor; those are the 2 most common contraindications. The other complicated issue is that this combination must be taken with a meal of at least 500 calories. It can’t be a protein bar or a shake; it really is supposed to be a meal. These might be reasons to not give this to our patient, but you could consider it.   What about (option A), abacavir/lamivudine/dolutegravir? With abacavir, some people are concerned about cardiovascular toxicity risk, but this is a 22-year-old guy; he’s not a smoker. We don’t have a history that makes us worried about that, so that’s not a bad combination.   Or would you choose (option G), tenofovir/emtricitabine/raltegravir? I wouldn’t. Raltegravir needs to be given twice a day. It probably could be given once a day, but that’s not the way it’s FDA approved. There was a study that showed that if the baseline viral load is &amp;lt; 100,000, people can do okay if they take it once a day. However, there’s probably no reason to give this to our patient.   I’m a proponent of single-tablet regimens. You can’t have adherence to just one part of the combination. You can’t have a mess-up at the pharmacy, where a patient goes and the pharmacy says, “Okay, I’ll give you this drug now, but we need to order that other drug, so come back in a couple of days.” Now they’ve got to go back, which is a hassle, and more problematic; a hurdle to adherence. Therefore, I think one of the combinations make good sense for this guy. Furthermore, since we were already worried that maybe this is an adolescent that won’t adhere and thus chose something convenient, we would also be worried that a bad adherence outcome would result in virologic rebound and virologic failure. In this scenario, I would be most worried about resistance as a consequence, not really a fallen CD4 count, so would give the combo that’s least likely to cause resistance in the event of virologic failure.
  18. HBsAg, hepatitis B surface antigen; HBV, hepatitis B virus; HCV, hepatitis C virus; WT, wild type. How would our choice change if our patient was HLA-B*5701 positive? What do we nix? Abacavir. What if he was hepatitis B surface antigen positive? You would want to use tenofovir. The TAF formulation would also work against hepatitis B, as shown in a study of HIV-coinfected patients presented by Joel Gallant at one of the large conferences sometime this past year. Finally, we already talked about why we wouldn’t want to give the rilpivirine, but anything else I think would be okay.   What if our patient was a woman? Some clinicians don’t like to give efavirenz to women. Efavirenz isn’t really one of our preferred initial agents anyway, so I don’t think it really makes as much difference. Drug–drug interactions are an important consideration, oral contraceptives being the one that would be the most potentially concerning.
  19. 3TC, lamivudine; ABC, abacavir; ART, antiretroviral therapy; ATV, atazanavir; BL, baseline; COBI, cobicistat; CrCl, creatinine clearance; DHHS, Department of Health and Human Services; DRV, darunavir; DTG, dolutegravir; EFV, efavirenz; eGFR, estimated glomerular filtration rate; EVG, elvitegravir; FTC, emtricitabine; HBsAg, hepatitis B surface antigen; RAL, raltegravir; RPV, rilpivirine; RTV, ritonavir; TDF, tenofovir disoproxil fumarate; VL, viral load. Here are the recommended guidelines from the DHHS, IAS-USA, and European AIDS Clinical Society. In general, the preferred regimens are now predominately the integrase inhibitor class plus tenofovir/emtricitabine, with the exception that abacavir/lamivudine is okay if you give it with dolutegravir. The DHHS is the only guideline to include TAF and to no longer include any NNRTIs. The European guidelines are a little bit more modern and, therefore, are more similar to the DHHS guidelines. Both include darunavir as their only boosted PI regimen; atazanavir is now off both lists. Because the IAS-USA guidelines are a little bit older—they’re from 2014—they still include the NNRTI combinations tenofovir/emtricitabine/efavirenz, abacavir/lamivudine with efavirenz, and tenofovir/emtricitabine/rilpivirine, as well as still include boosted atazanavir in addition to boosted darunavir. Furthermore, TAF is not included because it wasn’t approved when those guidelines came out. Why are these the preferred agents? We will use the next group of slides to explore how various studies have shaped current guidelines.
  20. ATV, atazanavir; BID, twice daily; CV, cardiovascular; DRV, darunavir; FTC, emtricitabine; RAL, raltegravir; RTV, ritonavir; QD, once daily; TDF, tenofovir disoproxil fumarate; TF, tolerability failure; VF, virologic failure. When the ACTG study was done, the question was: What, other than an efavirenz-based regimen, is the next best way to approach therapy for a patient starting out? At the time, atazanavir and darunavir were the 2 preferred protease inhibitors and raltegravir was the available integrase inhibitor. Thus, the options were tenofovir/emtricitabine with either boosted darunavir, boosted atazanavir, or raltegravir.
  21. ATV, atazanavir; DRV, darunavir; RAL, raltegravir; RTV, ritonavir; VF, virologic failure. In this complicated analysis, raltegravir did the best from a tolerability perspective. Darunavir did a little bit better than atazanavir, again from a tolerability perspective, which isn’t surprising. The main toxicity associated with atazanavir is hyperbilirubinemia, which turns people&amp;apos;s eyes yellow and occurs with enough frequency that it gets them to say, “Hey, I don&amp;apos;t want to walk around trying to explain to people why my eyes are yellow, take me off of this drug.” Thus, for that reason, boosted darunavir outperforms boosted atazanavir.
  22. ATV, atazanavir; COBI, cobicistat; d/c, discontinuation; EVG, elvitegravir; FTC, emtricitabine; RTV, ritonavir; TDF, tenofovir disoproxil fumarate. Study 103 is one of the pivotal phase III registrational trials for tenofovir/emtricitabine/elvitegravir/cobicistat, a comparison to tenofovir/emtricitabine plus boosted atazanavir, showing slightly better outcomes in the folks who were getting elvitegravir/cobicistat. The difference was not statistically significant but noninferiority was observed.
  23. ART, antiretroviral therapy; ATV, atazanavir; COBI, cobicistat; eGFR, estimated glomerular filtration rate; EVG, elvitegravir; FTC, emtricitabine; TDF, tenofovir disoproxil fumarate; QD, once daily; RTV, ritonavir; Tx, treatment. The WAVES study compared elvitegravir/cobicistat/emtricitabine/TDF to tenofovir/emtricitabine/boosted atazanavir in a population that was almost exclusively women with at least a viral load of &amp;gt; 500 copies. This is actually the largest study done exclusively in women in the antiretroviral study armamentarium and was undertaken because so few women were included in initial phase III studies of elvitegravir/cobicistat/emtricitabine/TDF.
  24. AE, adverse event; ATV, atazanavir; BL, baseline; BMD, bone mineral density; COBI, cobicistat; eGFR, estimated glomerular filtration rate; EVG, elvitegravir; FTC, emtricitabine; HDL, high density lipoprotein; LDL, low density lipoprotein; TDF, tenofovir disoproxil fumarate; RTV, ritonavir. The results of the WAVE study showed that patients did a little bit better on elvitegravir/cobicistat/emtricitabine/TDF than they did on boosted atazanavir. However, you may be wondering why they included boosted atazanavir instead of boosted darunavir when they sort of knew that there would be more discontinuations in the atazanavir arm due to the known tolerability issue (ie, yellowing of the eyes due to hyperbilirubinemia). The reason is that there was some data supporting the safe use of boosted atazanavir in pregnancy and they wanted to allow women who became pregnant in the course of the study to continue and not drop out. Perhaps boosted darunavir as a head-to-head comparator would have been more of a fair comparison, because you wouldn&amp;apos;t have had patients dropping out because of hyperbilirubinemia. There were similar numbers of virologic failures in each arm of this study.
  25. 3TC, lamivudine; ABC, abacavir; DTG, dolutegravir; EFV, efavirenz; FDA, US Food and Drug Administration; FTC, emtricitabine; QD, once daily; TDF, tenofovir disoproxil fumarate; Tx, treatment. There have been a series of studies in treatment-naive patients demonstrating the benefit of dolutegravir, including SINGLE, SPRING-2, and FLAMINGO. SINGLE compared dolutegravir plus abacavir/lamivudine vs tenofovir/emtricitabine/efavirenz, which at that point was the gold-standard single-tablet regimen. Results showed better outcomes in people who were treated the dolutegravir-inclusive regimen vs the efavirenz-based regimen. Although these data clearly show there was superiority of abacavir/lamivudine plus dolutegravir compared to the efavirenz-based combination, it appears to largely be due to tolerability issues.
  26. AE, adverse event; BID, twice daily; DTG, dolutegravir; RAL, raltegravir; QD, once daily. SPRING-2, another key dolutegravir study, compared dolutegravir vs raltegravir, the first available integrase inhibitor. Again, outcomes appeared to favor dolutegravir.
  27. ART, antiretroviral therapy; DRV, darunavir; DTG, dolutegravir; RTV, ritonavir. Finally, FLAMINGO compared dolutegravir vs boosted darunavir, with dolutegravir achieving better outcomes compared to boosted darunavir. Again, the outcomes here aren&amp;apos;t really differences in virologic failure rates, they are more differences in tolerability.
  28. COBI, cobicistat; eGFR, estimated glomerular filtration rate; EVG, elvitegravir; FDA, US Food and Drug Administration; FTC, emtricitabine; TAF, tenofovir alafenamide fumarate; TDF, tenofovir disoproxil fumarate. What about the new TAF formulation? This question was tackled by studies 104/111 in a head-to-head comparison of elvitegravir/cobicistat/emtricitabine/tenofovir alafenamide vs elvitegravir/cobicistat/emtricitabine/tenofovir DF.
  29. AE, adverse event; BMD, bone mineral density; COBI, cobicistat; EVG, elvitegravir; FDA, US Food and Drug Administration; FTC, emtricitabine; TAF, tenofovir alafenamide fumarate; TDF, tenofovir disoproxil fumarate. As you can see here, the head-to-head of elvitegravir/cobicistat/emtricitabine/tenofovir alafenamide vs elvitegravir/cobicistat/emtricitabine/tenofovir TDF basically resulted in a tie. Arithmetically, people do 2% better on TAF vs TDF; however, this difference is not statistically significant but rather is noninferior.
  30. BMD, bone mineral density; COBI, cobicistat; EVG, elvitegravir; eGFR, estimated glomerular filtration rate; FTC, emtricitabine; TAF, tenofovir alafenamide fumarate; TDF, tenofovir disoproxil fumarate. The advantage of the new TAF formulation is that there is less renal toxicity. eGFR goes down in both the TDF and TAF arms, just not quite as much in the people who get the TAF formulation. Cobicistat is known to decrease the distal renal tubular secretion of creatinine, so it’s not a surprise that a small decline in eGFR is observed in both arms. Presumably, though, TDF is contributing to this in some way, but I’m not sure exactly how. The teaching is that we don’t see early renal toxicity from tenofovir, so I’m curious if some drug–drug interaction is causing the effect, that we’re seeing more of the effect of the cobicistat in the presence of TDF. This difference is seen in both black and nonblack patient populations.  
  31. ART, antiretroviral therapy. The advantages of single-tablet regimens are obvious: convenience, no partial adherence, fewer copays, which are increasingly an issue for my patients. However, I’m not sure that it’s fair to say that single-tablet regimens not being available for every kind of regimen is a disadvantage. The world isn’t perfect. We’re fortunate to have as many good single-tablet regimen choices as we do. What more do we want?
  32. ART, antiretroviral therapy; DHHS, Department of Health and Human Services; FDA, US Food and Drug Administration. These are the single-tablet regimens that we currently have available. It really is amazing how convenient they are. When I’m rounding on our inpatient HIV service—we have an inpatient HIV service that mostly serves HIV patients with their non-HIV problems—and I tell the residents that I would much rather have an HIV infection than diabetes, they look at me like I’m some kind of freaky old guy. But in response, I explain that treatment of diabetes has become so complicated that I don’t even know how to manage it anymore. Its treatment requires all sorts of oral medications, 5 different kinds of insulin, and you’ve got to prick your finger and monitor yourself. With HIV, you can take just 1 pill a day, which is easy. You can live a normal life expectancy if you take your medications every day. I believe that. Although the data suggest that people with HIV don’t have a normal life expectancy, it’s because there’s a whole group of people that present very late in the game or don’t adhere to their regimens. The majority of people who maintain virologic suppression and adhere to the therapy are going to live to be nice old men and women.
  33. ART, antiretroviral therapy; COBI, cobicistat; CrCl, creatinine clearance; DHHS, Department of Health and Human Services; EVG, elvitegravir; FTC, emtricitabine; TAF, tenofovir alafenamide fumarate. Data support early initiation of therapy, even in people with CD4 counts &amp;gt; 500. The guidelines say treat everybody irrespective of CD4 count. The DHHS guidelines no longer recommend NNRTI regimens for antiretroviral initiation. Additionally, elvitegravir/cobicistat/emtricitabine/tenofovir alafenamide has been shown to be safe in patients with a creatinine clearance as low as 30—people with renal dysfunction and CrCl of 30, whether occurring while on TDF or non-tenofovir–inclusive regimens, were shown to have no change in renal function while on elvitegravir/cobicistat/emtricitabine/tenofovir alafenamide, at least over a relatively short period of time (ie, 6 months to a year). 
  34. DM, diabetes mellitus; NSTEMI, non-ST segment elevation myocardial infarction; PCP, primary care physician. A 53-year-old man with multiple medical problems, including diabetes and hyperlipidemia, is newly diagnosed with HIV infection by his primary care provider. He was hospitalized 2 years ago with an NSTEMI. He has been doing better in the last year, sticking to his medical regimen, rarely missing a dose of his prescribed medications: metformin, glipizide, aspirin, metoprolol, and atorvastatin. He used to smoke; doesn’t do any illicit drugs, never did.
  35. ALT, alanine aminotransferase; ART, antiretroviral therapy; AST, aspartate aminotransferase; BUN, blood urea nitrogen; Cr, creatinine; DLV, delavirdine; DM, diabetes mellitus; eGFR, estimated glomerular filtration rate; EFV, efavirenz; HBV, hepatitis B virus; HCV, hepatitis C virus; LDL, low density lipoprotein; NPV, nevirapine. His CD4 count is 423; viral load about 70,000. His HIV genotype is K103N, which is resistant to efavirenz, nevirapine, and delavirdine. He is HLA-B*5701 negative, his creatinine is 1.6, and his estimated GFR is 53. His liver function tests are normal; he is HBV immune. His last hemoglobin A1c was 6.5—not terrible, but could be better. His LDL is 105. Thus, his hyperlipidemia and diabetes are relatively well controlled. Upon hearing the news of his HIV infection, he says, “Okay, tell me what to do, doc. I’ll do whatever you say.”
  36. BUN, blood urea nitrogen; Cr, creatinine; eGFR, estimated glomerular filtration rate; GT, genotype; HBV, hepatitis B virus; HCV, hepatitis C virus; LDL, low density lipoprotein; NSTEMI, non-ST segment elevation myocardial infarction. What NRTI combination would you recommend for this guy? Recall that his eGFR is 53 and his creatinine is 1.6. He has diabetes, hyperlipidemia, and cardiovascular risk. He had a NSTEMI 2 years ago.   Would you choose abacavir/lamivudine? You might be concerned about cardiovascular risk in people who get abacavir, and we will look at some data supporting that concern.   How about tenofovir DF/emtricitabine? You could use it, but it’s not an ideal choice. Why? You can’t give it every day, which could be a real problem for some patients. Even I don’t like to do things some days out of the week but not other days of the week because, honestly, I can never remember what day it is. I would surely mix up having to take a medication one some days, but not others. I like to do the same thing every day.   How about elvitegravir/cobicistat/emtricitabine/tenofovir alafenamide? I think this is a reasonable combination, single-tablet regimen; in fact, it’s the only single-tablet regimen that you could use in this kind of patient.   How about a different NRTI combination? Would anybody want to use stavudine/didanosine or zidovudine/lamivudine? No, a different NRTI combination is not the best choice.     Would you use lamivudine or emtricitabine without other NRTIs? This is an interesting question. I would say that although you may not feel comfortable with that option right now, you may be doing more of that in the future.   Finally, how about not using NRTIs at all in this patient? Because there’s some renal insufficiency, you may very well want to consider a NRTI-sparing regimen. However, I’ve been having problems getting elvitegravir/cobicistat/emtricitabine/tenofovir alafenamide—for example, it’s either not on the insurance company’s formulary—so I’ve had to write a medical reason why. But in some cases, I can’t even get it authorized. Sometimes it just takes a while before these newer drugs get on the formularies.
  37. BUN, blood urea nitrogen; Cr, creatinine; eGFR, estimated glomerular filtration rate; GT, genotype; HBV, hepatitis B virus; HCV, hepatitis C virus; LDL, low density lipoprotein; NSTEMI, non-ST segment elevation myocardial infarction. Which other agents would you use in the regimen? Of course, this would depend on which primary regimen you chose. If you chose elvitegravir/cobicistat/emtricitabine/tenofovir alafenamide, you’re already using an integrase inhibitor. If you chose an NRTI-sparing regimen, you may choose to add a boosted PI plus an integrase inhibitor, which I think would be the most common NRTI-sparing approach. However, maybe instead you would choose a boosted PI and one NRTI, which I&amp;apos;ve never done in my clinical practice, but perhaps is a possibility I should be thinking about.  
  38. CV, cardiovascular; DM, diabetes mellitus; HTN, hypertension. Here we see common comorbidities in HIV patients who are on Medicare and are over 65 vs a younger Medicaid population. Not surprisingly, we are seeing growing numbers of HIV patients with diabetes, hypertension, kidney disease, and cardiovascular disease. Our patients with HIV are getting older.
  39. ABC, abacavir; ART, antiretroviral therapy; CVD, cardiovascular disease; FPV, fosamprenavir; HCV, hepatitis C virus; LPV, lopinavir; RTV, ritonavir; TDF, tenofovir disoproxil fumarate. The IAS-USA recommends avoiding abacavir, lopinavir, and ritonavir as initial ART in patients with a high risk for cardiovascular disease. They also warn against boosted fosamprenavir, which I didn’t realize carried an increased risk for cardiovascular events. It could just be that there is more data for atazanavir and darunavir that don&amp;apos;t show an increased risk of cardiovascular events, so why not use the agents where we have a little bit more reassurance? In patients with reduced renal function, TDF should generally be avoided, especially in combination with a boosted PI. Remember that the boosted PI increases tenofovir concentrations, so if we&amp;apos;re worried about renal risk from tenofovir, we don&amp;apos;t want to, in effect, give a higher dose of the tenofovir. TDF, especially with a boosted PI, should also be avoided in patients with an elevated fracture risk (eg, HCV coinfection, postmenopausal women, or individuals who already have osteoporosis).
  40. ARV, antiretroviral; ATV, atazanavir; CKD, chronic kidney disease; LPV, lopinavir; RTV, ritonavir; TDF, tenofovir disoproxil fumarate. The D:A:D study demonstrated an increased risk of chronic kidney disease due to antiretroviral therapy. It is well known that tenofovir is associated with an increased risk of renal dysfunction but so are atazanavir and lopinavir/ritonavir. In fact, in an adjusted analysis, the risk of chronic kidney disease for atazanavir is similar, if not higher, than that associated with tenofovir. Although you probably don’t have a lot of patients on lopinavir/ritonavir these days, you may have patients on boosted atazanavir. For these patients, please remember that atazanavir causes renal stones and is associated with an increased risk of renal toxicity. Furthermore, if you observe an increase in creatinine in a patient on tenofovir-boosted atazanavir, you may want to exchange the boosted atazanavir for an integrase inhibitor (in the absence of resistance) but keep them on tenofovir. 
  41. ABC, abacavir; CC, case-control; CV, cardiovascular; CVD, cardiovascular disease; MI, myocardial infarction; RCT, randomized controlled trial; TDF, tenofovir disoproxil fumarate. 1. Friis-Moller N, et al. Eur J Cardiovasc Prev Rehabil. 2010;17:491-501. 2. Friis-Moller N, et al. Eur J Prev Cardiol. 2015;[Epub ahead of print]. 3. SMART/INSIGHT Study Group. AIDS. 2008;22:F17-24. 4. Martin A, et al. Clin Infect Dis. 2009;49:1591-1601. 5. Durand M, et al. J Acquir Immune Defic Syndr. 2011;57:245-253. 6. Obel N, et al. HIV Med. 2010;11:130-136. 7. Choi AI, et al. AIDS. 2011;25:1289-1298. 8. Young J, et al. J Acquir Immune Defic Syndr. 2015;69:413-421. 9. Rotger M, et al. Clin Infect Dis. 2013;57:112-121. 10. Palella F, et al. CROI 2015. Abstract 749LB. The D:A:D study also demonstrated an association of abacavir with an increased risk of cardiovascular disease, a finding that has since been corroborated by a whole host of studies. However, the most recent data from the NA-ACCORD group shows an equivocal rate of cardiovascular risk in treatment-naive people who are starting on abacavir combinations.
  42. ABC, abacavir; CC, case-control; CV, cardiovascular; MI, myocardial infarction; OR, odds ratio; RCT, randomized controlled trial; TDF, tenofovir disoproxil fumarate. 1. Lang S, et al. Arch Intern Med. 2010;170:1228-1238. 2. Ribaudo HJ, et al. Clin Infect Dis. 2011;52:929-940. 3. Bedimo RJ, et al. Clin Infect Dis. 2011;53:84-91. 4. Ding X, et al. J Acquir Immune Defic Syndr. 2012;61:441-447. 5. Palella F, et al. CROI 2015. Abstract 749LB. There are also a host of studies that don’t support a cardiovascular risk with abacavir. The one that’s probably cited the most often is the FDA analysis. The FDA did their own meta-analysis of individuals who got abacavir or didn’t get abacavir in the context of randomized clinical trials and did not find an increased risk of MI in the people who got abacavir. Now keep in mind that these were mostly younger individuals, with relatively low risk for cardiovascular disease. Because in almost every antiretroviral therapy study conducted in naive patients, the median age of the population is 38—if anybody ever asks you what the median age of the population is in an antiretroviral-naive study, always say 38 because that is the right answer.   You are probably going to be more likely to see cardiovascular events or other rare events in large cohorts than you are in randomized clinical trials where the duration of the trial is relatively short. However, in the NA-ACCORD study, where everybody who was on abacavir was considered, including people who were naive to therapy or switching to abacavir, there still was not a statistically significant increased risk for MI associated with abacavir. It’s definitely a topic for debate. My approach is to avoid abacavir in patients with cardiovascular risk because why not? Why give them something where there may be risk? Why not give them something where you know there is not a risk? And we know that tenofovir is not associated with a risk.  
  43. BMD, bone mineral density; COBI, cobicistat; eGFR, estimated glomerular filtration rate; EVG, elvitegravir; FTC, emtricitabine; TAF, tenofovir alafenamide fumarate; TDF, tenofovir disoproxil fumarate. In a subanalysis of study 104/111, which compared elvitegravir/cobicistat/emtricitabine/TDF and elvitegravir/cobicistat/emtricitabine/tenofovir alafenamide in people 50 years of age or older, again shows a slightly more beneficial outcome for the TAF formulation, but one that is not statistically significant but instead is noninferior.
  44. CKD, chronic kidney disease; COBI, cobicistat; DM, diabetes mellitus; eGFR, estimated glomerular filtration rate; EVG, elvitegravir; FTC, emtricitabine; HTN, hypertension; QD, once daily; TAF, tenofovir alafenamide fumarate; TDF, tenofovir disoproxil fumarate. Here we see a large phase III trial of individuals with chronic kidney disease who switched to elvitegravir/cobicistat/emtricitabine/TAF from a variety of regimens, including elvitegravir/cobicistat/emtricitabine/TDF, tenofovir/lamivudine/efavirenz, or tenofovir/emtricitabine/boosted atazanavir.
  45. COBI, cobicistat; eGFR, estimated glomerular filtration rate; EVG, elvitegravir; FTC, emtricitabine; TAF, tenofovir alafenamide fumarate; TDF, tenofovir disoproxil fumarate. For patients who switched to elvitegravir/cobicistat/emtricitabine/TAF, we saw a slight improvement in renal function, in particular in people that were not on TDF at baseline. However, the results were dependent on which formula was used for calculating the eGFR or the creatinine clearance. Regardless, it is clear that kidney function did not get worse after the switch, which was supported by improvements in urinary protein markers following the switch.
  46. COBI, cobicistat; CrCl, creatinine clearance; EVG, elvitegravir; FTC, emtricitabine; TAF, tenofovir alafenamide fumarate; TDF, tenofovir disoproxil fumarate. Elvitegravir/cobicistat/emtricitabine/TAF is FDA approved down to a creatinine clearance of 30. Elvitegravir/cobicistat/emtricitabine/TDF is not recommended when the creatinine clearance is &amp;lt; 70.
  47. 3TC, lamivudine; ABC, abacavir; BUN, blood urea nitrogen; Cr, creatinine; DTG, dolutegravir; eGFR, estimated glomerular filtration rate; FTC, emtricitabine; GT, genotype; HBV, hepatitis B virus; HCV, hepatitis C virus; LDL, low density lipoprotein; NSTEMI, non-ST segment elevation myocardial infarction; TAF, tenofovir alafenamide fumarate; TDF, tenofovir disoproxil fumarate. If you were considering an abacavir-, tenofovir-, or TAF-sparing regimen, which would you most strongly consider? Honestly, I wouldn&amp;apos;t consider a sparing regimen in this type of patient.
  48. BL, baseline; DRV, darunavir; FTC, emtricitabine; PDVF, protocol-defined virologic failure; RAL, raltegravir; RTV, ritonavir; TDF, tenofovir disoproxil fumarate. The NEAT study compares boosted darunavir plus raltegravir to boosted darunavir plus tenofovir/emtricitabine. In patients with failure—treatment failure or virologic failure, which is more common in the people who’d get the integrase inhibitor and the boosted PI—there is not a statistically significant difference; the 95% confidence interval crosses 0. However, in patients whose baseline viral loads are ≥ 100,000, it really does look like they do better on the 2 NRTIs plus boosted darunavir vs the NRTI-sparing regimen.  
  49. 3TC, lamivudine; AE, adverse event; ART, antiretroviral therapy; D/C, discontinued therapy; FTC, emtricitabine; LPV, lopinavir; RTV, ritonavir; Tx, treatment. The GARDEL study is one of the more head-scratching studies that I’ve seen in the last couple of years. It compares lopinavir/ritonavir plus lamivudine vs lopinavir/ritonavir plus lamivudine or emtricitabine plus another NRTI. What combination is going to do better? Will 3 drugs going to do better than 2 drugs? Because we’ve been preaching to medical students since forever that you’ve have to use 3 drugs to treat people with HIV, they all should answer, “Yeah, 3 drugs are going to do better than 2.” However, in the GARDEL study, higher virologic suppression rates were observed for patients who received just 2 drugs, one NRTI and the boosted PI, a benefit that was maintained for 96 weeks without more virologic failure. How do you explain that? I don’t know how you explain that, but that is, in fact, the data.
  50. 3TC, lamivudine; BL, baseline; DTG, dolutegravir; FDA, US Food and Drug Administration; HBsAg, hepatitis B surface antigen; IQR, interquartile range; ITT-e, intent to treat-exposed; QD, once daily. I think the PADDLE study, which was presented at the 2015 European AIDS Conference, is a really amazing study. The PADDLE study evaluated the use of dolutegravir plus lamivudine in treatment-naive patients. There were 2 cohorts of 10 patients. They first tested the combination in 10 patients to see how well it worked. When they confirmed it worked well in those 10 patients, they added another 10 patients. They choose their patients carefully, including only patients with a viral load of &amp;lt; 100,000 and a CD4 count ≥ 200, presumably in an effort avoid choosing a patient population that would be more likely to fail. Although some people whose baseline viral loads were &amp;gt; 100,000 made it into the study. Every single person in this study ended up getting virologic suppression by 8 weeks and maintained it through Week 24.   Again, another example where virologic success was achieved by 2 drugs, but this time by an integrase inhibitor plus an NRTI, not an integrase plus a boosted PI. What does that mean? It means this integrase inhibitor has a relatively high genetic barrier to resistance. Is it as good as a boosted PI? I don’t know, but it is very intriguing data. A number of the integrase inhibitors are showing a higher genetic barrier to resistance, so although we need more comparative studies and more data with regard to durability of the genetic barrier, we may be treating people with 2 drugs in more creative ways in the future. It’s certainly not ready for prime time, I wouldn’t yet prescribe this regimen in any patients, but I would let one of my patients into a study that was testing it if the patient was carefully selected (eg, somebody with renal dysfunction and cardiovascular disease that didn’t want to be on a boosted PI) and followed closely.  
  51. ABC, abacavir; ART, antiretroviral therapy; BID, twice daily; DRV, darunavir; LPV, lopinavir; RAL, raltegravir; RTV, ritonavir; TDF, tenofovir disoproxil fumarate. Here are the recommendations for the use of NRTI-sparing regimens. Regimens using fewer than 2 NRTIs should only be used in patients who cannot take abacavir or tenofovir, including lopinavir/ritonavir plus lamivudine twice a day or boosted darunavir plus raltegravir. However, boosted darunavir plus raltegravir should be used only for patients whose baseline viral loads are &amp;lt; 100,000 and CD4 counts are &amp;gt; 200.   Remember, the DHHS guidelines: They don’t go out on a limb; they make recommendations based upon data. You might ask why not maybe use boosted darunavir plus lamivudine? Although that would be a rational thought, there is no study that looks at boosted darunavir plus lamivudine. The DHHS guidelines will not endorse a combination where there is no data, even if it makes sense based upon what we understand about the drugs studied in other comparative ways.
  52. ART, antiretroviral therapy; CT, computed tomography; ED, emergency department; OI, opportunistic infection. A 40-year-old woman presents to the emergency department with left arm weakness, difficulty walking, and dysarthric speech. She has a prior history of HIV, but she&amp;apos;s homeless and has neither entered into care for her HIV nor received antiretroviral therapy. She has a frontoparietal mass with extensive edema. Toxoplamsa IgG is positive. She is started on sulfadiazine, pyrimethamine, and folinic acid to improve her neurologic status.
  53. HBsAg, hepatitis B surface antigen; HBV, hepatitis B virus; HCV, hepatitis C virus. Her lab results show a viral load of 175,000 and CD4 count of 65. Her genotype test is pending, but she is coinfected with HCV and is HLA-B*5701 negative. He has no known history of heart or renal disease. She does have a history of intravenous drug use, smokes a pack a day, and does drugs when she can get her hands on them.
  54. ART, antiretroviral therapy; GT, genotype; HBsAg, hepatitis B surface antigen. When would you recommend starting her on antiretroviral therapy? If you chose option B, to defer ART until she completes treatment for toxoplasmic encephalitis, when exactly would that be? When would toxoplasmic encephalitis treatment be done? Answer, never, so probably not the right answer. If you chose option C or D, to defer antiretroviral therapy until she’s in a stable living situation or until she’s in a drug treatment program, that also might not be the best choice because I think a case can be made that this woman with a very complicated psychosocial history and that still may be doing drugs is a very high risk of loss to follow-up. So maybe the best choice is to make sure she shows up in your office before starting therapy because we’ll do her no good if we start her on therapy before she leaves the hospital and she never comes back. I think requesting that she makes a presence in your office before starting her on therapy is not an unreasonable choice.    
  55. ART, antiretroviral therapy; Crypto, Cryptosporidium; OI, opportunistic infection; OR, odds ratio; PCP, Pneumocystis pneumonia. The ACTG 5164 study addressed the question of immediate ART therapy in patients with acute opportunistic infections (ie, starting ART therapy 2 weeks after their OI diagnosis) vs waiting for 8 weeks (ie, starting after individuals have gotten a longer duration of their treatment for their OI, presumably out of concerns about IRIS). Results of the study showed that new AIDS-defining events were less likely to occur in people who started therapy early versus waiting. Most of the people in the study had Pneumocystis; there were fewer bacterial infections. Other opportunistic infections included 13 cases of toxoplasmosis, Cryptococcus, and other fungal infections.
  56. ART, antiretroviral therapy; DDI, drug–drug interaction; DHHS, Department of Health and Human Services; OI, opportunistic infection; PCP, Pneumocystis pneumonia. Here are the DHHS-recommended guidelines for starting ART in patients with selected opportunistic infections. For Pneumocystis, it is recommended to start ART within 2 weeks. For toxoplasmosis, most clinicians would start ART within 2-3 weeks. However, this is based upon a relatively limited data set. For Mycobacterium tuberculosis, the recommendation is to start within 2 weeks if the CD4 count is &amp;lt; 50 but by 8-12 weeks for all other patients. However, I think most clinicians would treat within 2 weeks. For Cryptosporidiosis, the recommendation is to start right away. Finally, for Cryptococcal meningitis, the recommendation is to delay therapy at least for 2 weeks. However, some people would wait for a longer period of time.
  57. ART, antiretroviral therapy; GT, genotype; HBsAg, hepatitis B surface antigen. What regimen would you start for our 40-year-old female patient that has high viral load and low CD4 count, is HLA-B*5701 negative and HCV positive, has a history of injection drug use, and has toxoplasmosis? Is there any one that you want to cross off your list? I see no reason to give her an efavirenz-based regimen. I think it’s reasonable to give her a single-tablet regimen, although we shouldn’t give her dolutegravir/abacavir/lamivudine because she has HBV. Thus, maybe elvitegravir/cobicistat/emtricitabine/TDF or elvitegravir/cobicistat/emtricitabine/TAF would be preferred. I think either is a reasonable approach, although you could make cases for other combinations. However, you don’t have her genotype test result back, so you might want to lean toward a combination with a lower genetic barrier to resistance, despite the fact that those regimens are more complicated. You can always readjust to something simpler if it turns out her virus isn’t resistant. You can explain that right now we’ve got to start on something that’s a little bit more complicated—not terribly—because we don’t want your virus to become resistant by giving you a regimen that we don’t know is fully active.
  58. BL, baseline; RT, reverse transcriptase; TAM, thymidine analogue mutation. This is the most recent data on the probability of transmitting resistant virus. The samples came from 4 phase III clinical trials of elvitegravir/cobicistat/emtricitabine/TDF. The results showed an increase in the baseline NNRTI resistance in the community—most commonly K103N mutations— as well as some NRTI and PI, but little integrase inhibitor resistance. There was also a big study that was done by the California Collaborative Group and Monogram Biosciences that didn’t find any integrase inhibitor resistance among 300 individuals who were naive to therapy. However, as we use more integrase inhibitors, we will likely find more integrase inhibitor resistance, but it tends to be not the key mutations that are associated with virologic failure.
  59. ART, antiretroviral therapy. In summary, we’ve got data for early initiation of treatment. We recommend therapy for everybody irrespective of their CD4 count. We’ve got some revised recommendations for what ART combinations to start; however, I still recommend some individualization of regimen choice based upon adherence and other characteristics of the patient’s disease at the time that they present. We have the advent of TAF for patients with chronic kidney failure and issues with bone mineral density, though I think the main advantage will be in the treatment of patients with bone mineral toxicity more so than renal. I say that, but I guess honestly I have a number of patients in their 60s and 70s with abnormal creatinine levels that I would feel more comfortable taking off tenofovir. Their creatinine levels fluctuate between 1.7 and 1.5 every other visit, and with it, my feeling is that I’ve got to get them off TDF. However, most of these patients are not candidates for elvitegravir/cobicistat/emtricitabine/TAF because of their complicated treatment histories; many of them are people that I’ve been taking care of for 15 or 20 years. TAF plus emtricitabine in a fixed-dose combination is the regimen I would need for many of these folks, and really I think it’ll be an important drug for many individuals with renal dysfunction. It’s not going to be the panacea, but it’ll be an aid.